You are on page 1of 47

NEPHROLOGY SAMPLEX

November 2020

1. The drug of choice in diabetic patients who present with


hypertension and proteinuria is
a. Diltiazem (calcium channel blocker) - For 6. Extreme acidemia can cause
tubulointerstitial disease & adult polycystic kidney a. Seizure
(e.g. Verapamil & Diltiazem) b. Diarrhea
b. Clonidine (alpha-2 agonist) c. Anemia
c. Losartan (angiotensin receptor blocker) d. Delirium
d. Atenolol (beta blocker)
Extreme acidemia:
The first line treatment in patients with Neurologic - lethargy, stupor, coma and
DM and HTN are ARB and ACEi seizure
Cardiovascular - arrythmia (ventricular
tachycardia), decreased
2. Which of the following conditions is associated with response to
muddy brown granular cast? epinephrine, both lead
a. NSAID associated Nephropathy to hypertension
b. Contrast Induced Nephropathy
c. Acute Tubular Necrosis - (+) parenchymal damage 7. RIFLE criterion that describes loss of renal function
d. Henoch-Schonlein purpura requires a persistent acute kidney injury or complete loss of
renal function for more than…
3. Pre-renal acute renal failure a. 2 weeks
a. Adequate renal plasma flow - inadequate renal b. 3 weeks
plasma flow and intraglomerular hydrostatic c. 4 weeks
pressure to support normal glomerular filtration d. 5 weeks
b. Decreased cardiac output
c. Normal intraglomerular hydrostatic pressure -
inadequate
d. Prolonged ischemic - prolonged prerenal can lead
to ischemic injury (ATN)

4. Which of the following is a finding of pre-renal azotemia?


a. FENa >1 - ATN
b. Urine osmolality >500
c. BUN/Creatinine ration <20
d. Specific gravity >1.020

8. Which of the following is considered a functional


biomarker?
a. Creatinine & BUN
b. Cystatin C
c. KIM-1 Tissue Biomarkers
d. NGAL

9. Normal oxygen saturation is


5. What is the recommended energy source for a CKD a. 95-100%
patient? b. 100-125%
a. 30 kcal/kg c. 90-95%
b. 35 kcal/kg d. 80-85%
c. 40 kcal/kg
d. 45 kcal/kg

Protein restriction - 0.6 - 0.75 g/kg/day


NEPHROLOGY SAMPLEX
November 2020

10. Protein restriction in CKD: 18. The safest diagnostic modality to assess renal size:
a. Increase erythropoietin production a. Ultrasound of the kidney
b. Reduced uremic symptoms b. CT Scan of the kidney
c. Increased renal blood flow c. IVP
d. Reduced renal osteodystrophy d. Magnetic resonance imaging (MRI)

Slow the rate of decline in 19. Which of the following causes a normal anion gap
early stages metabolic acidosis?
Reduce uremic symptoms a. Rhabdomyolysis
b. Diabetic ketoacidosis
11. Which of the following is seen in patients with CKD who c. Lactic acidosis
present with Malnutrition - Inflammation Atherosclerosis d. Renal tubular acidosis
Calcification (MIAC) syndrome?
a. Decrease in C-reactive protein - increased
b. Increase serum albumin - decreased
c. Decrease in fetuin
d. Increase in glucagon

12. Which of the following drugs can lead to impaired renal


autoregulation?
a. NSAIDs & ACEi
b. Amphotericin B
c. Aminoglycosides
d. Cisplatin

13. In diabetic hypertensive with proteinuria of >1gm/24hr,


BP should be reduced to what level?
a. 110/80
20. Where is the best site to take arterial blood gas
b. 130/80
sample?
c. 160/90
a. Femoral artery
d. 140/90
b. Arterial catheter
c. Radial artery
14. Which of the following is not associated with intrinsic
d. Central line
AKI?
a. Diarrhea
21. What clinical findings will point to chronic
b. Sepsis
glomerulonephritis?
c. Ischemia
a. Hypertension
d. Nephrotoxins
b. Anemia
c. Azotemia
15. The following are causes of anemia in CKD, except:
d. Bilaterally contracted kidneys
a. Iron deficiency
b. Folate deficiency
c. Chronic inflammation Hypertension
d. Hypoparathyroidism Renal insufficiency
Proteinuria >3 g/day
16. Which of the following abnormal endocrine functions is Shrunken smooth kidneys
observed in CKD?
a. Decrease half-life of insulin 22. Which of the following golmerular disease has the
b. Increased tolerance to electrolyte loading highest chance of recurrence and graft failure in
c. Decrease erythropoietin production transplanted kidney?
d. Increase vitamin D activation a. Membranoproliferative GN type 1
b. Lupus Nephritis
17. What is the first step in the management of a patient c. Anti GBM Disease
with acute hyponatremia d. FSGS - 50%
a. Assess renal function
b. Assess volume status
c. Assess urine electrolytes
d. Assess thyroid function
NEPHROLOGY SAMPLEX
November 2020

23. Which is NOT a trigger for mesangial matrix expansion 28. What is most likely diagnosis of a 20 y/o male who
in DM glomerulopathy? consulted the OPD for recurrent episodes of gross
a. Increase myoinositol hematuria, with unremarkable PMH and family history.
b. Advanced glycosylated end products BP=110/70, (-) edema, (-) proteinuria, rbc=TNTC?
c. Direct effect of hyperglycemia on mesangial matrix a. IgA Nephropathy
d. Release of growth factors b. TBMD
c. Alport’s syndrome
d. Henoch Schoenlein Purpura
IgA Nephropathy
Normal BP
Gross hematuria
Dysuria
Edema and nephrotic syndrome - less frequent

29. Clinically, what is the difference between Acute


Glomerulonephritis and RPGN?
a. Oliguria
b. Edema - Acute glomerulonephritis
24. What hereditary nephritis does not results from c. Tea colored urine with hematuria
primary abnormalities in type IV collagen? d. Renal failure over days to weeks
a. Nail-Patellar Syndrome
b. IgA Nephropathy 30. What is the drug use to reduce proteinuria by reduction
c. TBMD on intraglomerular pressure?
d. Alport’s Syndrome a. Calcium channel blockers
b. Beta-blockers
25. What glomerular disease presenting as nephrotic c. Prostglandins
syndrome commonly have renal vein thrombosis as d. ACEI
complication?
a. FSGS - decreased risk 31. What is the significance of doing electron microscopy of
b. DM Nephropathy - decreased risk tissues from renal biopsy?
c. Membranous GN a. Assess the degree of glomerulosclerosis
d. Minimal Change Disease - decreased risk b. Detect presence and kind of immune deposits
c. Clarify principal location of immune deposits
26. Besides hematuria and proteinuria, what are other d. It will detect cellular proliferation
clinical features of Alport’s Syndrome?
a. Common to have mental retardation 32. Which of the following clinical situations will NOT
b. Glaucoma warrant renal biopsy in adult patient?
c. Dysplasia of dorsal limbs a. Isolated proteinuria of 3.5gms/24hrs
d. Sensorineural deafness b. Hematuria w/o proteinuria but with elevated
serum creatinine
c. Hematuria with proteinuria (+) 3
d. Isolated hematuria without proteinuria

33. What is modified WHO classification based on expected


renal biopsy findings in a 36 y/o female who was admitted
for generalized edema of 2 months duration, associated by
fever and arthralgia, maculopapular rashes on the face,
27. Which of the following statement is true with regards BP=160/100, grade (+)4 edema, pale, serum creatinine=1.8
to relation of microalbuminuria and Diabetes Mellitus? mg/dL, (+) anti dsDNA?
a. There is no relation of CV events and a. Class II
microalbuminuria in Type 1 DM b. Class III
b. Type 2 DM and microalbuminuria succumb to CV c. Class IV
events before they progress to clinical proteinuria d. Class V
c. Development of microalbuminuria is predictable e. Class VI
in type 2 DM
d. It should be tested at time of diagnosis of Type 1
DM
NEPHROLOGY SAMPLEX
November 2020

34. What will differenitate Henoch-Schoenlein purpura 40. Which glomerular disease will NOT have
from IgA Nephropathy who presents with similar immunofluorescence finding on renal biopsy of
glomerular syndrome and histopathologic findings? predominant mesangial IgA deposits?
a. Developed after URTI a. IgA Nephropathy
b. Normal serum IgA level b. DM Nephropathy
c. Glomerular syndrome responsive to steroids c. Henoch-Schoenlein Purpura
d. Clinical findings of arthralgia, abdominal pain and d. Lupus Nephritis
purpura.
41. What is the severity of hyperkalemia in a 60 y/o female,
35. A 20 y/o female was admitted for edema and oliguria. diabetic, with serum potassium=6.8 meq/L and ECG changes
She complained of on and off fever with sorethroat 3 weeks of multifocal PVC’s?
PTA. BP=160/100, with periorbital edema. Urinalysis a. Mild
revealed proteinuria=(+)1, rbc = 20-30hpf. What serological b. Critical
test will give you the most likely diagnosis? c. Moderate
a. ANCA d. Severe
b. Anti GBM titer
c. ASO titer
d. ANA titer

36. What special staining is use to detect collagen deposits


and degree of glomerulosclerosis and interstitial fibrosis?
a. Jones-Methenamine stain - rnhance basement
membrane structures
b. Hematoxylin and Eosin stain - assess celllularity 42. What will be the expected effect on cardiovascular
and architecture system in a 42 y/o female with CKD stage 5 with laboratory
c. Masson’s trichrome stain results of serum creatinine = 4.8 mg/dL, serum potassium =
d. Congo red stain - amyloid deposits 4, pH = 6.92, pCO2 = 19, HCO3 = 8?
a. Pulmonary edema
37. What is NOT part of management in a 22 y.o male b. Hypotension
smoker with oliguria, tea-colored urine and hemoptysis, c. Arrythmias
BP=160/110, periorbital edema, Lungs with rales mid to d. Tachycardia
base bilateral, serum creatinine=8.7mg/dL, proteinuria (+2),
rbc=25-30/hpf, (+)ANCA (+)AntiGBM? 43. What condition can give rise to either SIADH or Central
a. Pulses of methylprednisolone Diabetes Insipidus?
b. Dialysis a. Hypothyroidism
c. Renal transplantation b. Primary Hyperaldosteronism
d. Plasmpheresis c. Encephalitis
38. What is the most common glomerular lesion in DM d. Analgesic abuse nephropathy
glomerulopathy?
a. Diffuse glomerulosclerosis 44. What is the major cation intracellular?
b. Thickened glomerular basement membrane a. Magnesium
c. Proliferative GN b. Sodium
d. Increase mesangial matrix with acellular nodules c. Potassium
d. Calcium
39. What is histopathologic findings in Acute
Post-streptococcal GN that will connote poorer prognosis? 45. What is the total deficit in a 62 y/o male, 70kg, who had
a. Presence of dome shaped deposits describe as lbm and vomiting for 3 days, with BP=palpatory 70mmHg,
“humps” - electron microscopy HR=120/min with dry lips and buccal mucosa?
b. Diffuse irregular, finely granular deposits of Ig on a. 3L
capillary wall b. 4L
c. Light microscopy findings of diffuse cellular c. 2L
proliferation - diffuse endocapillary proliferative d. 5L
GN
d. Immunofluorescence findings of garlan pattern -
starry sky pattern
NEPHROLOGY SAMPLEX
November 2020

46. Which is NOT true with regards to tubular absorption of 52. What is the type of RTA in a 32 y/o male presenting
water and sodium? with lower extremity weakness with urinalysis: pus cells=
a. Proximal tubule is impermeable to sodium and 8-15 hpf, pH=7.0, serum potassium=2.8 meq/L, ABG:
water pH=7.20, pCO2= 19, HCO3 = 10 and renal UTZ with
b. Descending loop of Henle is impermeable to nephrocalcinosis?
water and permeable to sodium - permeable to a. Type 1
water, impermeable to sodium b. Type 2
c. Collecting ducts is permeable to water with ADH c. Type 3
and permeable to sodium with aldosterone d. Type 4 (??)
d. Ascending loop of Henle is permeable to water and
impermeable to sodium 53. To expand the extracellular compartment what IV fluids
will you use?
47. What diuretic causes metabolic alkalosis and a. 0.9% NaCl (dapat NSS and not D5W)
hypokalemia? b. 5% Dextrose in Water
a. Furosemide c. Albumin
b. Spironolactone d. Dextran
c. Amiloride
d. Hydrochlorthiazide 54. What clinical condition will not dictate urgent
treatment of Hypokalemia? - admission to hospital,
48. What will be the immediate treatment in a 68 y/o with continuous cardiac monitoring, immediate treatment
serum potassium = 6meq/L, and ECG findings of ventricular a. Acute myocardial infarction
tachycardia? b. Hypokalemic patient for emergency surgery
a. D50-50 plus Regular Insulin c. Serum potassium <3meq/L
b. Dialysis d. Hepatic encephalopathy
c. Calcium gluconate e. Presence of cardiac arrhythmia
d. Salbutamol nebulization
55. What body fluids have highest potassium content?
49. What can cause normal anion gap metabolic acidosis? a. Small intestinal fluids
a. ESRD b. Colonic fluids
b. Type 2 RTA - and diarrhea c. Saliva
c. COPD in acute exacerbation d. Gastric fluids
d. Ethylene glycol toxicity
56. What will NOT be included on the differential diagnosis
50. What factor have no effect on the development of signs of a 52 y/o female with laboratory results of serum
and symptoms in 65 y/o hypertensive noted to have serum potassium = 3.0, AG= 10, pH = 7.14, pCO2 = 20, HCO3 = 14?
sodium = 110 meq/L a. Distal RTA
a. Other associated abnormalities in the brain b. Proximal RTA
b. Rate of decline in serum sodium c. CKD stage 5
c. Degree of changes in serum sodium d. Cholera - (+) diarrhea and vomiting
d. The age of the patient
57. What condition will NOT cause both hyponatremia and
51. What clinical findings will differentiate moderate hypernatremia?
volume depletion from severe volume depletion? a. Vomiting and diarrhea
a. Dry mucous membrane - mild b. Primary hyperaldosteronism -hypernatremia only
b. Resting tachycardia c. Mannitol administration
c. Flat neck vein d. Encephalitis
d. Hypotension
58. What etiology of Hypokalemia is NOT due to
Resting tachycardia and Postural hypotension intracellular shift of potassium?
To differentiate moderate from mild a. Metabolic Alkalosis
To differentiate severe from moderate b. Beta blockers
c. Hyperthyroidism
d. Salbutamol
e. Insulin
RHEUMATOLOGY SAMPLEX
November 2020

1. Which of the following joint is generally spared in


R.A.?
a. Distal interphalngeal 6. Which of the following organism is unikely to cause
b. Wrist reactive arthritis?
c. Cervical a. Staphylococcus
d. Proximal interphalangeal b. Chlamydia
c. Campylobacter
Rheumatoid Arthritis affects small joints than big joints
PIP, MCP, Intercarpal and carpometacarpal Shigella
Salmonella
2. K.L 38 y/o female, consulted because of pain on the Yersenia
right foot, more severe after taking the first step in the Campylobacter
morning upon waking up. Pain is usually lessens with Chlamydia trachomatis
weight bearing activity during the day but worsens with
continued activity. What is your diagnosis? 7. L.A. 30 year old male with psoriasis of 10 years,
a. Retrocalcaneal bursitis - inflammation of bursa consulted because of hand arthritis of 3 months. PE
at the back of the heel bone showed (+) salmon colored hyperkeratotic scaling
b. Ankle arthritis plaques on the hands, pitting and oncycholysis of nails,
c. Plantar fasciitis - pain in the first step in arthritis of the 2nd, 3rd, and 4th digits of both hands.
morning, (+) tenderness What is your diagnosis?
d. Achilles tendonitis a. Reactive arthritis
b. Hand osteoarthritis
Retrocalcaneal bursitis - assoc. With gout, c. Psoriatic arthitis
Rheumatoid arthritis, trauma. d. Rheumatoid arthritis
Spondyloarthitides
Plantar fasciitis - obesity, pes planus(flat 8. A.J. 28 y/o female consulted because of arthritis of
foot), pes cavus (high-arched foot), limited hands, wrist, metacarpophalangeal, and elbow joints
dorsiflexion of the ankle, prolonged standing, for 3 months, there is morning stiffness of 4 hours.
walking on hard surfaces What laboratory test would you request to confirm
your diagnosis?
3. Drug induce lupus is associated with intake of: a. Erythrocyte sedimentation rate
a. ACE inhibitors b. Anti-cyclic citrullinated peptide - more specific
b. Hydralazine - Procainamide, Phenytoin for RA
c. Simvastatin c. MRI
d. Ciprofloxacin d. C reactive protein (CRP)

4. Which serologic marker for R.A maybe seen in 9. Schirmer’s test is performed in patients suspected
affected patients years before the onset of clinically with:
apparent disease and is more specific for the diagnosis a. Sjogrens syndrome - test for tear secretion
of RA? b. Mixed connective tissue disease - Reynaud’s
a. C-reactive protein phenomonen
b. Rheumatoid factor - nonspecific c. SLE
c. Anti-cyclic citrullinated peptide d. Systemic sclerosis
d. ASO titer
10. The most common neuropsychiatric manifestation
5. Bacteria enters the joint space from the blood of SLE:
stream because of: a. Migraine
a. Large blood volume pass through synovial b. Depression
vessels c. Psychosis
b. Absence of limiting basement membrane of d. Cognitive dysfunction
synovial capillaries
c. Heavy microbial load in the blood
d. Poor quality of the cartilage

Bacteria adhere to articular cartilage


Cartilage destruction (48h)
RHEUMATOLOGY SAMPLEX
November 2020

11. S. 35 yo male athlete, 5’5ft 140lbs consulted 17. Among the connective tissue disease, this has the
because of left ankle arthritis of 1 day. He exercised in highest mortality:
the gym for 6h and claims to have drank Gatorade after a. MCTD
the exercise. Pain score was 10/10 and the joint is b. Sjogrens syndrome
warm, tender, swollen, with limitation of motion. c. Dermatomyositis
WHAT IS YOUR DIAGNOSIS? d. Systemic sclerosis
a. Gout
b. Fracture 18. A connective tissue disease with overlapping
c. Sprain features of Scleroderma, SLE, and polymyositis is
d. Septic arthitis known as:
a. Mixed connective tissue disease
12. M.L 23 yo call center agent, consulted because of 2 b. Undifferentiated CTD
month history of arthritis involving the ankle, MCP, c. Fibromyalgia syndrome
shoulder, and elbows. The arthritis tends to be d. NONE
migratory, symmetric and non erosive. A month later
she noted photosensitivity, fever and cough. Which test 19. What autoantibody test is most specific for active
would you request to strengthen your diagnosis? - SLE, nephritis and vasculitis?
infectious arthritis a. AntiSm
a. ANA b. AntiJo
b. CRP c. AntidsDNA
c. CBC d. ANA
d. Rheumatoid gout
20. The dermatologic lesion seen in patients with
13. The first line treatment of patients with Ankylosing Dermatomyositis include:
spondylitis is: a. Heliotrope rash - upper eyelid
a. NSAIDS b. Mechanics hand
b. Paracetamol c. Gottrons papules - most common
c. Opioid derived drugs d. AOTA
d. Glucocorticoids
21. Most common etiologic agent of bacterial arthitis:
14. What is the characteristic of the monosodium urate a. E.coli - 3rd
crystal under polarized microscopy? - Gout b. Neisseria sp.
a. Rectangular, positively birefringent c. S. Aureus
b. Globular, non birefringent d. Streptococcus pneumoniae - 2nd
c. Needle shaped, negatively birefringent
d. Rhomboid, negatively birefringent 22. Bony hyperthrophy of the distal interphalangeal
joint is known as:
15. Which of the inflammatory myopathies is not a. Bouchards node - Proximal interphalangeal
responsive to glucocorticoids? b. Swan neck deformity
a. Inclusion body myositis c. Heberdens node
b. Polymyositis d. Oslers node
c. Dermatomyositis
d. NONE 23. This rheumatologic condition usually presents as
chronic inflammatory oligoarthitis
16. Hyperextension of the first interphalangeal joint a. Rheumatoid arthitis
with palmar sublaxation of the first b. Septic arthritis
metacarpophalangeal joint will manifest as: - c. Reactive arthritis
Rheumatoid arthritis d. Gouty arthritis
a. Swan neck deformity - felxion of DIP &
hyperextension of PIP joint 24. Which of the following statement is not true
b. Piano key deformity regarding osteoarthritis?
c. Boutonnier deformity - flexion of PIP & a. More common in men than women - female
hyperextension of DIP joint is more common
d. Z-shaped thumb b. Joint subjective to repetitive use are prone to
develop osteoarthritis
c. Common in weight bearing joint
RHEUMATOLOGY SAMPLEX
November 2020

d. Obesity increases the risk 30. The diagnosis of osteoarthritis is most likely in a 60
yo patient with right knee pain if:
25. Finkelstein test will be positive with this a. White cell count of synovial fluid is 700
periarticular disorder: cells/hpf
a. Carpal tunnel syndrome - (+) Tinel’s sign b. Sugar level of synovial fluid is 90mg.ml
b. Iliopsoas bursitis c. Blood uric acid is 320 mg/dl
c. De Quervains tenosynovitis d. Protein content of synovial fluid is 50mg/ml
d. Medial epicondylitis - golfers elbow /
resistance to wrist flexion 31. An inflammatory joint problem is more painful or
symptomatic in what situation?
26. What do you call the condition where one or more a. During activity
fingers of the hand can lock or snap into a bent position, b. After exposure to cold temperature
causing pain and lack of function on attempted c. After rest or sleep
straightening? This involves inflammation of the d. At bedtime
sheaths of the flexor digitorum tendons.
a. Carpal tunnel syndrome 32. The most common site of spinal Tuberculosis in
b. De Quervains tenosynovitis adult: - thoracolumbar spine
c. Lateral epicondylitis a. Sacral vertebra
d. Stenosing tenosynovitis - trigger finger b. Low cervical
c. Lower thoracic and upper lumbar
27. A 36 yo healthy male, sexually active developed d. Upper thoracic
fever, malaise with acute right knee pain, swelling and
decreased range of movement. His synovial fluid gram 33. How long will you give antibiotic therapy in this
stain showed Gram-positive cocci in clusters. Which of particular patient?
the following microorganism is the most probable a. 4 wks
cause? b. 6 wks
a. Escherichia sp c. 1 wk
b. Pseudomonas sp d. 2 wks
c. Streptococcus sp
d. Staphylococcus sp 34. A 36 yo healthy male, sexually active developed
fever, malaise with acute right knee pain, swelling and
28. What do you call this condition when the joint decreased range of movement. His synovial fluid gram
capsule adheres to the humeral neck causing restricted stain showed Gram-positive cocci in clusters. The best
active and passive motions in all planes? empiric therapeutic option for this case is:
a. Frozen shoulder - adhesive capsulitis a. Penicillin
b. Bicipital tendinitis - anterior shoulder pain b. Ceftriaxone
c. Rotator cuff tendinitis - pain in abduction c. Vancomycin
d. Impingement syndrome d. Piperacillin-tazobactam

29. A 46 yo farmer, diabetic complained of a wound 35. Which statement is true of pseudogout?
over his hand. This started as a small, red, painless a. More common in female - males = females
nodule that progressed into an ulcerated lesion that b. Commonly affects the metatarsophalangeal
spreads in a line following the draining lymphatic joint - affects knee, hand, polyarthricular
channels forming nodules on the skin along the way. He c. Maybe precipitated by severe illness, trauma
later complained of left ankle joint pains and swelling or surgery
with skin abscess. What is the most probable fungal d. ALL
infection implicated in this patient?
a. Cryptococcosis 36. A patient with arthritis of the knee commonly
b. Blastomycosis assumes the position of:
c. Histoplasmosis a. External rotation
d. Sporothricosis b. Extension
c. Abduction
d. Flexion
RHEUMATOLOGY SAMPLEX
November 2020

37. A patient was diagnosed with De-Quervains 43. Which of the following drugs oxidize uric acid to
tenosynovitis what maneuver will you do to test if the allantoin?
patient is positive for the said condition? a. Febuxostat - allopurinol / xanthin oxidase
a. Drop arm test - rotator cuff tendinitis inhibitor
b. Phalen’s test - carpal tunnel syndrome b. Probenecid - uricosuric drugs
c. Finkelstein’s test c. Anakinra
d. Tinel’s test - carpal tunnel syndrome d. Pegloticase

38. A.L, 40 yo female vendor consulted because of left 44. What do you call the bony enlargement found on
knee arthritis of 7 days. No history of trauma, but a the distal interphalangeal (DIP) joint in patients with
history of fever and cough was present. PE (+) bulge osteoarthitis?
sign, warm, tender, with limitation of movement was a. Bouchard’s node - PIP
noted. What is the best diagnostic test for her b. Osteophytes
condition? c. Heberden’s node
a. Xray of both knees d. Tophi - gout
b. Blood uric acid determination
c. Synovial fluid analysis 45. An 80 y/o grandmother complained of painful
d. Chest xray knees more pronounced on walking. On physical
examination she was noted to have bilateral genu
39. The features of fibromyalgia include diffuse varum, (+) crepitations both knee joints, (+) non-tender
tenderness at discrete anatomical sites and: bony nodules on the proximal and distal
a. Polyarthritis interphalangeal joints of the 2nd and 3rd digits. If this is a
b. Sleep disturbance case of Osteoarthritis, the pathologic sine qua non of
c. Chest pain this condition is
d. Fever a. Outgrowth of osteophytes
b. Sclerosis of subchondral bony plate
40. An acute attack of gout needs to be distinguished c. Weakness of muscles bridging the joint
from: d. Hyaline articular cartilage loss and articular
a. Pseudogout cartilage loss
b. Cellulitis
c. Septic arthitis 46. This is one of the commonly affected joints in this
d. ALL condition
a. Ankle joint
41. A complete deficiency of the enzyme b. Wrist joint spared in OA
hypoxanthine-guanine phosphoribosyl transferase c. Elbow joint
(HGPRT) where patients present with severe neurologic d. 1st metatarsal phalangeal joint
findings, mental retardation developing gout or renal
stones because of the persistent hyperuricemia is called 47. ____provides the cartilage its tensile strength
a. Glycogen storage disease type III a. Hyaluronic acid
b. Kelley seegmiller b. Matrix metalloproteinase
c. Lesch-Nyhan syndrome c. Type 2 collagen
d. Glycogen storage disease type 1 d. Aggrecan - provides compressive stiffness

42. In the treatment of early stage Lyme disease


arthritis for how long will we give the antibiotic?
a. 20 days
b. 30 days
c. 7 days
d. 14 days
RHEUMATOLOGY SAMPLEX
November 2020

48. Arthrocentesis of the right knee joint effusion was


done. Which of the following synovial fluid finding is 54. Which of the following is considered a risk factor
seen in osteoarthritis? for Pseudogout?
a. White count <1,000/uL a. Alcoholism
b. Polymorphonuclears >50% b. Hypocalcemia
c. Polymorphonuclears >75% - septic arthritis c. Hyperparathyroidism
d. Hypermagnesemia
d. white count >2,000/uL - inflammatory, RA,
Gout
55. What do you call the bony enlargemtn found on
the proximal interphalangeal (PIP) joint in patients with
osteoarthritis
a. Osteophytes
b. Heberden’s nodes - DIP
c. Tophi
d. Bouchard’s nodes

56. If the tissue biopsy will grow Candida sp. What is


49. Chondrocalcinosis is usually seen in what type of
the antifungal treatment of choice for Candida
crystal induced arthropathy?
arthritis?
a. Pseudogout
a. Amphotericin B
b. Gouty arthritis
b. Penicillin
c. Oxalate arthropathy
c. Nystatin
d. Calcific tendinitis
d. Itraconazole
50. Prophylaxis treatment is considered if the vector is
57. A 55 yo female comes to you complaining of 2 days
identified immediately. Which of the collowing drugs is
history of left knee joint pain with swelling, warmth
the recommended prophylactic therapy for this
and erythema of the involved joint. Which of the
patient?
following conditions would you most likely consider?
a. Rifampicin
a. Rheumatoid arthritis
b. Amoxicillin
b. Osteoarthritis
c. Ceftriaxone
c. Psoriatic arthritis
d. Doxycycline
d. Crystal induced arthropathy
51. Xray of the knee was likewise requested. Which of
58. What important differential diagnosis should be
the following radiographic finding is seen in
excluded in acute gouty attack involving a single knee?
osteoarthritis?
a. Bacterial arthritis
a. Bony spurs
b. Osteoarthitis - monoarticular
b. Rat bite appearance - gout
c. Pseudogout
c. Overhanging edges - gouty arthritis
d. Calcium pyrophosphate disease
d. Chondrocalcinosis - pseudogout
59. Which type of crystal deposition triggers
52. Which among these spirochete causes Lyme
pseudogout?
disease arthritis?
a. Monosodium urate - Gout
a. Leptospira sp
b. Calcium pyrophosphate dihydrate
b. Borrelia sp
c. Calcium oxalate - CADD
c. Treponema sp
d. Basic calcium phosphate
d. Spirochetae sp
60. In varus deformity the stress placed across the ___
53. This is a radiographic finding in patients with gouty
compartment of the knee joint.
arthritis
a. Lateral
a. Overhanging edges
b. Lower
b. Bone fusion in the joint
c. Inner
c. Chondrocalcinosis - pseudogout
d. Medial
d. Double contour sign - seen in UTZ
RHEUMATOLOGY SAMPLEX
November 2020

Articular characterized by
A. crepitus, deformities, swollen 18. Destruction of cartilage in septic arthritis occur
B. pain with activity within _ hrs
C. point tenderness A. 24 hours
D. pain at passive and active motion B. 48 hours
C. 72 hours
7. Consider as most common type of arthritis: D. 96 hours
A. Gouty Arthritis
B. Osteoarthritis 20. Infectious arthritis due to S. aureus are treated with
C. Rheumatoid Arthritis oxacillin, nafcillin, and vancomycin are treated for
D. Infectious Arthritis A. 1 week
B. 2 weeks
8. Joint protectors EXCEPT: C. 3 weeks
A. Afferent Sensory D. 4 weeks
B. Joint Capsule
C. Synovial Fluid 22. Rheumatoid Arthritis
D. Bursa A. Acute Polyarticular
B. Chronic Polyarticular
10. Pain in osteoarthritis arises from the following except C. Acute Monoarticular
(sorry, can't remember yung order ng choices) D. Chronic Monoarticular
A. Capsular Stretching
B. Cartilage Loss 23. A patient complaining of reactive symmetric form of
C. Synovitis polyarthritis with visceral or disseminated tuberculosis is
D. Bone Marrow Erosion/Effusion diagnosed with what disease?
A. Poncet’s disease
B. Reiter’s syndrome
12. Initial analgesic of choice in patients with OA of C. TB arthritis
knees, hip and ankle joints D. Charcot disease
A. NSAID
B. Prednisone 24. Reactive arthritis is common in young male, it is
C. Acetaminophen linked to what potential genetic predisposing factor?
D. Opiate HLA-B27

13. Provides cartilage its tensile strength. #26-30 : Matching Type


A. Aggrecan 26.Enthesitis - inflammation of the attachment site of
B. metalloproteinase tendon or muscle to bone
C. Type 2 collagen 29. Epicondylitis - Inflammation or infection involving the
D. Hyaluronic acid epicondyle
30. Subluxation - Incomplete alignment of joints
14. Valgus deformity stress is placed on the_____ 34. Most common cause of bacterial arthritis
compartment of the knee joint. Staph aureus
A. Inner 36. Etiologic agent in Lyme disease:
B. Medial A. Borrelia Burgdorferi
C. Lower B. Listeria Monocytogenes
D. Lateral C. Neisseria Gonorrhoeae
D. Staphylococcus Aureus
15. Most common organism causing arthritis in
adolescents and young adults 37. TB arthritis manifested as
- Neisseria sp. A. Polyarticular joints
B. Large weight bearing joint
17. What type of arthritis syndrome is present as a C. Fulminant infection
chronic monoarticular arthritis D. Acute onset
A. Septic Arthritis
B. Gouty Arthritis 38. Which of the following is the characteristic of
C. Rheumatoid Arthritis articular disorder?
D. Osteoarthritis A. Swelling, Deformity is present
RHEUMATOLOGY SAMPLEX
November 2020

B. Presence of Focal or Point Tenderness D. Antiphospholipid Antibody Syndrome (APAS)


C. Physical Finding is remote on Joint Capsule
D. Pain on active movement but without pain on 53. Schirmer’s test is performed in patients suspected
passive with:
A. Sjogren's Syndrome
39. lab results can be seen in OA except: B. Systemic Lupus Erythematosus
A. Hyperalbuminemia C. Osteoarthritis
B. Thrombocytosis D. Rheumatoid Arthritis
C. Anemia of Chronic Illness
D. Elevated ESR 54. Most Common Pulmonary Symptom of SLE:
A. Cough
41. The goal of the physical examination of the joints B. Dyspnea
A. Functional consequence of the process C. Pleuritic Chest Pain
B. Identify the nature of underlying process D. Hemoptysis
C. Identify involved structures
D. All of the above 55. Male, drinking spree for 8 hours last night. Most
appropriate treatment
A. Allopurinol
44. BT 25 y/o call center agent, pain in ankle, MCP, B. Increased water intake
shoulder and elbows. One month prior to consultation, C. Massage
she experienced photosensitivity and fever? What will D. Cold compress
you request to strengthen your diagnosis?
A. ANA
B. dsDNA 57. An Acute Gouty Attacks should be distinguished from
C. CBC which of the following/s?
D. —— A. Pseudogout
B. Cellulitis
45. Most Common Pulmonary Symptom of SLE: C. Septic Arthritis
A. Cough D. All of the Above
B. Dyspnea
C. Hemoptysis 59. RR, 25 year old female consulted her obstetrician
D. Pleuritic Chest Pain because of 3 episodes of unexplained abortion before 10
weeks of gestation. Her platelet is 90,000/dl. She had a
49. The signs/symptoms manifest early in systemic transient ischemic attack 6 months ago. What is your
sclerosis may precede development of other features by diagnosis?
months to years A. Dengue
A. Dyspnea B. SLE
B. Arthritis C. Thrombocytopenic Purpura
C. Raynaud’s Phenomenon D. Antiphospholipid Antibody Syndrome (APAS)
D. ——
60. Extra Articular manifestation of Osteoarthritis:
51. A connective tissue disorder with overlapping A. Renal
features of Scleroderma, SLE, Polymyositis is known as: B. Cardiac
A. fibromyalgia syndrome C. GIT
B. Undifferentiated CTD D. No extra articular manifestation
C. Mixed Connective Tissue Disease
D. None of the above

52. RR, 25 year old female consulted her obstetrician


because of 3 episodes of unexplained abortion before 10
weeks of gestation. Her platelet is 90,000/dl. She had a
transient ischemic attack 6 months ago. What is your
diagnosis?
A. Dengue
B. SLE
C. Thrombocytopenic purpura
CARDIOLOGY SAMPLEX
November 2020

d. Heart Transplant
1. A patient was admitted with BP of 220/120 mmHg with e. Eplerinone
left side weakness fundoscopy revealed Papilledema. He is
in what stage of hypertension 6. The Gold standard in the diagnosis of heart failure is
a. Hypertensive Emergency which of the following.
b. Hypertension Stage 1 - 140/90 - 150/99 a. NT PRO BNP
c. Hypertensive Urgency b. ECG
d. Hypertension Stage 2 - >160/100 or more c. 2D echo with doppler
d. Nuclear imaging
2. 12 lead ECG findings of ST elevation in Lead I, AVL, V5-V6
a. Lateral Wall 7. 50 male patient is in chronic heart failure and had been
b. Infero Lateral Wall taking Digitalis and Aldactone with out improvement from
c. Diffuse or extensive anterior wall his easy fatigue and shortness of breath. BP=100/60 HR=70.
d. Antero Lateral Wall 2D echo revealed Eccentric left ventricular hypertrophy with
e. Inferior wall ejection fraction = 25%. What will be the best drug for this
patient.
a. Trimetazidine - inc. oxygenation
b. Nebivolol - inc vasoldilator
c. Furosemide - for dyspnea
d. Ramipril - ACE inhibitor
e. Sacubitril/Valsartan - ARB

8. Renal sodium retention in hypertension leads to


resetting in the presence of hypertension. Which of the
following is observed?
a. Decrease renal vascular resistance
b. Decrease in peritubular oncotic pressure
c. Increase in peritubular hydrostatic pressure
d. Increase in filtration fraction

9. 60 y/o female diabetic admitted because of severe


heaviness of the chest with cold sweating. The ECG revealed
elevated ST in I & AVL - V5&V6 troponin T-1200 ng/mL
3. A 60 years old male, office manager, obese, complained a. Ticagrelor (Brilinta)
of severe chest. He was brought immediately to a nearby b. Fondaparinux
medical center. Her Bp was 180/110 mmHg. ECG was done c. Rivaroxabam
and revealed ST depression with T wave inversion in I and d. Streptokinase
AVL-V5 & V6 Troponin I revealed 600 n.g. e. Bivalirudin direct thrombosis inhibition
a. Chronic stable angina pectoris (Chronic C.A.D.)
b. Unstable angina pectoris 10. A patient diagnosed with congestive heart failure
c. STEMI myocardial infarction suffered from PND and Orthopnea. Which will be the initial
d. NSTEMI myocardial infarction treatment for the patients whose lung finding is acute
pulmonary edema.
4. Considered superior to clopidogrel and indicated to a. Nebivolol
reduce the rate of C.V death. It is used for ACS for the 1st 12 b. Isosorbide Dinitrate
months. c. ARB - or ACE
a. Ticagrelor (Brilinta) d. Flurosemide
b. Bivalirudin direct thrombosis inhibition e. Digoxin
c. Fondaparinux
d. Rivaroxabam 11. A post menopausal female 52 y/o, Asymptomatic,
e. Streptokinase would like to know her BP. Her mother is hypertensive at 45
y/o. Upon examination her BP is 130/90 mmHg on both
5. 48 male with refractory heart failure NYHA class III-IV arms. What will you do to this patient.
(Stage D) 2D Echo Eccentric Left ventricular hypertrophy a. Submit patient for ECG
with EF=20% already on saccubitril Valsartan, Furosemide, b. Submit patient for 2D echo doppler
Carvedilol, Aspirin, Atorvastatin. What else can you offer? c. Give antihypertensive drug
a. CABG d. Advice 24 hr ambulatory BP monitoring
b. CRT
c. Primary coronary intervention
CARDIOLOGY SAMPLEX
November 2020

12. 12 lead ECG findings of ST elevation in Leads V1-V3. d. Finrous plaque


What is your diagnosis?
a. Lateral wall 19. Which of the following murmur increases in intensity
b. Antero Lateral wall with inspiration?
c. Inferior wall a. Tricuspid regurgitation
d. Anteroseptal wall b. Aortic stenosis
e. Infero Lateral wall c. Mitral regurgitaion
d. Pulmonic stenosis
13. The following are ECG finding in acute coronary e. MVP
syndrome, Except:
a. Presence of Q waves 20. Which of the following statements is TRUE regarding
b. ST elevation management of bleeding in a patient with venous
c. ST depression thromboembolism.
d. New complete right bundle branch block a. Give Andexamet Alpha IV for bleeding 2 to
e. New complete left bundle branch Rivaroxaban
b. Give Vitamin K IV for bleeding 2 to Dabigatran
14. A 50 year old female diabetic complained of chest c. Give Idraxucimab IV for bleeding 2 to Apixaban
heaviness with cold sweating. BP was 90/60 mmHg. ECG d. Give Cipraparantag IV for bleeding 2 to
revealed ST depression in II, III, AVF and V5&V6. Troponin T Fondaparinoux
revealed 0-50 ng. e. Give Vitamin K IV for bleeding 2 to low molecular
a. NSTEMI myocardial infarction weight heparin
b. Chronic stable angina pectrois (Chronic CAD)
c. STEMI myocardial infarction 21. Which of the following is/are left sided failure
d. Unstable angina pectoris symptom?
a. Neck vein engorgement - right
15. ECG revealed QS with inverted T in V1-V6, I and avL b. Hepatomegaly - Ascites
a. Inferior wall c. Orthopnea - PND & dyspnea
b. Lateral wall d. Bipedal edema - right
c. Infero Lateral wall
d. Diffuse or extensive anterior wall 22. Differential diagnosis of Mitral stenosis: Orthopnea is
e. Antero Lateral wall more likely present:
a. Mitral stenosis
16. A 42 y/o male patient diabetic, dyslipidemic, b. Left atrial myxoma
hypertensive, complained of chest heaviness everytime he c. Neither
climbs the stair of their office. The discomfort was relieved d. Both
by rest.
a. STEMI myocardial infarction 23. Arthritis in acute rheumatic fever is characterized as:
b. Unstable angina pectoris a. Involves the distal interphalangeal joint
c. NSTEMI myocardial infarction b. Non migratory
d. Chronic stable angina pectoris (chronic CAD) c. Symmetrical
d. Involves the knee, elbow & wrist joint.
17. A 60 y/o female was admitted because of easy
fatigability chest discomfort. BP 140/90 HR-120/min. Lung 24. 60 y/o hypertensive, diabetic, 30 pack yrs smoker
revealed bilateral crackles (+) jugular vein discomfort & complained of right leg and foot pain while walking for
bilateral lower extremity edema. 2D echo showed more than 200 meters and is relieved by rest. Physical
concentric left ventricular hypertrophy with adequate wall examination showed. BP=180/100 HR=100 RR=20. Normal
mation with EF=60% rate and rhythm, skin over the bilateral lower extremities
a. Heart failure with preserved ejection fraction are fair in color, warm to touch. Pulses right Brachial +2,
b. Heart failure with mid ranges EF Radial +2, Ulnar +2, Femoral +2, Popliteal +2, Posterior
c. Heart failure with reduced ejection fraction Tibial +1, Dorsalis pedis +1; left brachial, radial, ulnar,
d. Diastolic heart failure inguinal, femoral, posterior tibial, dorsalis pedis all +2. Your
initial impression is Peripheral Arterial Disease, what is the
Normal EF= 56-77% recommended initial non invasive diagnostic exam that will
confirm your diagnosis?
18. Which of the following is the initial lesion of a. Ankle Brachial Index
atherosclerosis? b. Pulse volume recording
a. Foam cells c. Arterial duplex scan of both lower extremities
b. Fibrous cap d. Arteriography
c. Fatty streak e. Toe Brachial index
CARDIOLOGY SAMPLEX
November 2020

25. A 32 y/o asymptomatic female has rapidly rising, 32. A 15 y/o female patient develops pleuritic chest pain,
forceful pulse that collapses quickly and with a BP 140/40. fever, and a friction rub heard at the lower sternal border.
The most likely diagnosis is The most likely involved portion of the heart
a. Aortic stenosis a. Myocardium
b. Mitral stenosis b. Pericardium
c. Mitral regurgitation c. Valves
d. Aortic regurgitation d. Myocardium
33. Which of the following symptoms of aortic stenosis has
26. Which of the following condition produces a continuous the poorest prognosis?
murmur? a. Syncope
a. Mitral stenosis b. Dyspnea
b. Aortic stenosis with aortic regurgitation c. Congestive heart failure
c. Tricuspid regurgitation d. Angina pectoris
d. Patent ductus arteriosus
34. Differential diagnosis of Mitral stenosis: Increase in the
27. Differential diagnosis of Mitral stenosis: Diastolic intensity of murmur on inspiration
murmur… a. Mitral stenosis
a. Mitral stenosis b. Tricuspid stenosis
b. Aortic regurgitation c. Both
c. Both d. Neither
d. Neither
Diastolic murmur: Aortic regurgitation, Pulmonic 35. 62 y/o female hypertensive and diabetic complained of
regurgitation, Tricuspid stenosis, Mitral stenosis right buttocks pain while walking for more than 200 meter
and is relieved by rest. If this is a case of peripheral arterial
disease, predict the location of the arterial stenosis.
a. Distal abdominal aorta
b. Right iliac artery
28. A patient with Valvular heart disease with less than c. Right dorsalis pedis artery
ordinary activity causes symptoms is classified based on d. Right anterior tibial artery
New York Heart Functional Classification e. Right femoral arterya
a. Class I
b. Class II 36. Which of the following is NOT a major Jones criteria?
c. Class III a. Subcutaneous nodules
d. Class IV b. Arthralgia
c. Chorea
29. Graham Steele murmur is a high pitched, decresendo d. Carditis
diastolic murmur heard along the left sternal border is e. Ertythema marginatum
associated with
a. Tricuspid regurgitation 37. A symptomatic 29 y/ female with a murmur is found to
b. Pulmonic regurgitation have mitral stenosis on echo. Which of the following
c. Aortic stenosis statement is TRUE of mitral stenosis?
d. Aortic regurgitation a. The murmur radiates to carotids
b. S3 may be present
30. A patient with mitral stenosis was noted to have a c. High pitch systolic murmur
heart rate of 120 bpm, BP = 100/60. Which of the following d. Associated with loud S1
agent has a negative chronotrophic effect with positive
inotrophic effect? 38. Which of the following treatment with degenerative
a. Digitalis calcific aortic stenosis slows progression of leaflet
b. ACE inhibitors calcification.
c. Non dihydropyridine calcium channel blockers
a. Beta blockers
d. Beta blockers b. Digitalis
c. ACE inhibitors
31. The click and murmur in a patient with MVP occurs
d. HMG-CoA reductase inhibitors
earlier with
a. Standing 39. Differential diagnosis of Mitral Stenosis: Left ventricular
b. Handgrip enlargement is present
c. Isometric exercise
a. Mitral stenosis
d. Squatting b. Mitral regurgitation
CARDIOLOGY SAMPLEX
November 2020

c. Both a. Catheter directed thrombolysis


d. Neither b. Systemic thrombolysis with rTPA
c. Dabigatran 150mg tab bid
40. 68 y/o hypertensive, diabetic, 35 pack yrs smoker d. Enoxaparin 0.6 cc SQ q12 plus Warfarin 5mg tab q
complained of right leg and foot pain while walking for hs
more than 200 meters and is relieve by rest. PE showed e. Fondaparinoux 2.5 mg sc od plus warfarin 5 mg tab
BP=180/100 HR=100 RR=20. Normal rate and rhythm, skin qhs
over the bilateral lower extremities are fair in color, warm
to tough. Pulses right brachial +2, Radial +2, Ulnar +2, 45. The peak incidence of Rheumatic heart disease is
Femoral +2, Popliteal +2, Posterior tibial +1, Dorsalis pedis a. 5-14 y/o
+1; left brachial, radial, ulnar, inguinal, femoral, posterior b. Persons more than 30 y/o
tibial, dorsalis pedis all +2. Your initial impression is c. In older adolescent
Peripheral Arterial Disease and is confirmed with diagnostic d. Between 25 & 40 years
exam. Besides smoking cessation, blood pressure control
and blood sure control how will you treat this patient? 46. 50 y/o male underwent partial hip replacement due to
a. Cilostazol 100mg tab bid, Rosuvastatin 20mg tab a left femoral neck fracture 2 to a fall. He is bedridden for
qhs the past 5 days. On the 4th post op day the resident on duty
b. Vorapaxar 2.5mg tab od, Simvastatin 40mg tab q noticed that the left thigh is markedly bigger than the right
hs with associated erythema and pain. There is no fever. PMH:
c. Warfarin 5mg tab qhs, Rosuvastatin 20 mg tab qhs (-) HTN (-) DM2. Family History: (-) CVA, HTN. PSH:
d. Aspirin 80mg tab od, Clopidogrel 75mg tab, od, unremarkable. PPE. BP=130/70 HR=90/min RR=20 WT=90 kg.
Pravastatin 40mg qhs (-) pallor, APex beat 5th ICSLMCL, (-) heave (-) thrill, regular
e. Aspirin 8-mg tab od, Atorvastatin 40mg tab qhs rate and rhythm, (-) murmurs. Left thigh 68cm diameter,
right thigh 63cm. Left thigh is hyperemic, tender to touch.
41. An echocardiogram of a 22 y/o female patient reveals The diagnostic test to confirmed the diagnosis of acute deep
mitral valve prolapse. The most common physical findings is vein thrombosis. What diagnostic exam will confirm your
a. Continuous murmur diagnosis of Deep vein thrombosis?
b. Mid systolic click a. CT scan with contrast of the right lower extremities
c. Diastolic rumble b. Venous duplex scan of both lower extremities
d. Absent S1 c. Venography
d. MRI
42. Which of the following strategy is secondary prevention e. D-dimer
of acute rheumatic fever?
a. Elimination of overcrowded housing 47. A loud S1 is associated with
b. Prophylaxis with benzathine penicillin every 4 a. Aortic stenosis
weeks b. Mitral regurgitation
c. Mitral valve stenosis with severe calcification c. MVP
d. Complete treatment of group A streptococcal d. Mitral stenosis
infection with antibiotics
48. The recommended secondary prophylaxis for acute
43. Which of the following condition would likely produce a rheumatic fever without carditis is
widened pulse pressure? a. For 10 years after the last attack or 21 years of age
a. Aortic stenosis b. Lifelong prophylaxis
b. Aortic regurgitation c. For 5 years after the last heart attack or 21 years
c. Mitral valve prolapse of age
d. Tricuspid stenosis d. For 10 years after the last attack or 40 years of age

44. 50 y/o male underwent partial hip replacement due to 49. Atrial Fibrillation
a left femoral neck fracture 2 to a fall. He is bedridden for
the past 5 days. On the 4th post op day the resident on duty
noticed that the left thigh is markedly bigger than the right
with associated erythema and pain. There is no fever. PMH:
(-) HTN (-) DM2. Family History: (-) CVA, HTN. PSH:
unremarkable. PPE. BP=130/70 HR=90/min RR=20 WT=90 kg.
(-) pallor, APex beat 5th ICSLMCL, (-) heave (-) thrill, regular
rate and rhythm, (-) murmurs. Left thigh 68cm diameter,
No P wave
right thigh 63cm. Left thigh is hyperemic, tender to touch.
Irregular
The diagnostic test to confirmed the diagnosis of acute deep
Slow
vein thrombosis. How will you treat this patient?
CARDIOLOGY SAMPLEX
November 2020

Atrial Flutter

51. A 60 y/o man came to ER complaining of chest


Saw tooth appearance tightness at rest. He is a known smoker for 20 yrs
P wave with no regular medical check up. His 12 lead ECG
More P wave than QRS
AR = 200-300/min
showed - Cannot be determined
P wave = > 300/min
52. Ventricular fibrillation
Ventricular Tachycardia No discernible P, QRS,
ST segments
Most common cardiac
arrest in MI and ischemia
No ventricular
contraction
Wide QRS complex
Sx:
No P wave
No pulse
Rapid, bizarre >0.10
No BP
sec)
No perfusion
Px may collapse

Normal sinus Rhythm Initial intervention: Defibrillate

53. Which of the following is the most common cause of


heart disease in children in developing countries?
a. Hypertensive cardiovascular disease
b. Rheumatic heart disease
c. Ischemic heart disease
d. Congenital

50. Sinus rhythm with PVC 54. Acute rheumatic fever is commonly seen in this age
Wide QRS group?
No preceeding P wave a. 1-12 mo
T wave opposite QRS b. 25-40 yo
Compensatory pause c. 5-14 yo
d. 1-5 yo

55. Acute rheumatic fever is caused by


a. Mycoplasma
b. Streptococcus
c. Staphyloccus
d. Pseudomonas

Sinus rhythm with PAC 56. Which of the following is a sign of Acute Rheumatic
P wave different fever?
configuration from sinus a. Nausea
beat b. Joint tenderness
Long PR interval c. Malar rash
QRS narrow d. Change in vision
CARDIOLOGY SAMPLEX
November 2020

57. Which of the major Jones criteria has the longest latent d. Arthritis, arthralgia elevated ESR
period?
a. Chorea - 3 months 66. The following are complications of acute rheumatic
b. Erythema marginatum fever EXCEPT
c. Arthritis a. Inflammation of heart muscle
d. Subcutaneous nodules - 2-3 wks b. Valve regurgitation
c. Inflammation of the conducting system
58. Most common clinical feature of ARF is d. Pulmonary hemorrhage
a. Chorea - <2 -30%
b. Subcutaneous nodules - 5% 67. Which of the following confirms the diagnosis of ARF
c. Polyarthritis - 60-75% a. CRP
d. Erythema marginatum - <5% b. ASO
c. ECG
59. Which of the following cardiac valves is commonly d. ESR
affected in acute rheumatic fever?
a. Pulmonic 68. A minor jones criteria for the diagnosis of ARF?
b. Tricuspid a. Erythema marginatum
c. Aortic b. Subcutaneous nodules
d. Mitral c. Carditis
d. Migratory arthralgia
60. The hallmark of rheumatic carditis is
a. Pericarditis 69. True regarding subcutaneous nodules:
b. Myocarditis a. Mobile lumps beneath the skin
c. 1st degree AV block b. Associated with chorea - carditis
d. Valvular damage c. Are tender - painless
d. Symmetrical - asymmetrical
61. Rheumatic heart disease is best diagnosed by
a. Auscultation 70. The development of cardiac involvement in patients
b. Echocardiography with recurrent throat infection due to streptococcus can be
c. ECG explained by which mechanism?
d. CXR a. Hypersensitivity reaction to viral products
b. Molecular mimicry
62. The most common form of joint involvement in acute c. Direct damage on the heart tissues by the
rheumatic fever is arthritis. The following described the organism
arthritis in Acute Rheumatic fever except d. Loss of tolerance to self antigen
a. Involves the large joint
b. Polyarthritis 71. 42 yo male diabetic hypertensive with sever progressive
c. Symmetrical chest pain lasting for more than 20 minutes ECG revealed.
d. Migratory ST elevation in V1-V4 with Trop-T (+) quantitative of
480ng/ml. What is your diagnosis?
63. The drug of choice for the treatment of arthritis in acute a. Unstable Angina Pectoris
rheumatic fever is b. NonSTEMI
a. Tramadol c. STEMI
b. Naproxen d. Chronic stable angina pectoris
c. Glucocorticoids e. TIETZE syndrome
d. Aspirin
72. 32 yo male smoker w family history of suddent cardiac
64. The following results would indicate inflammation in a death (father at age 42) complaining of chest pain upon
patient with acute rheumatic fever except: climbing the stairs of the LRT. The discomfort is relieve upon
a. Elevated erythrocyte sedimentation rate resting. ECG revealed to be normal. What is your diagnosis?
b. Decreased hemoglobin & hematocrit a. TIETZE Syndrome
c. Elevated C reactive protein b. Chronic stable angina pectoris
d. Elevated WBC count c. Unstable Angina Pectoris
d. Non STEMI
65. According to the Jones criteria, which of the symptoms e. STEMI
and laboratory result would indicate a diagnosis of ARF
a. Fever, carditis, elevated WBC 73. Sudden chest pain and pallor after coming out of the
b. Arthritis, chorea, positive ASO titer bathroom. She is obese, hypertensive and diabetic.
c. Elevated CRP, ESR, carditis a. TIETZE
CARDIOLOGY SAMPLEX
November 2020

b. NONSTEMI
c. STEMI
d. Chronic Angina pectoris
e. Unstable Angina Pectoris

74. BP 180/110/ (+) diabetic, obese, smoker, and hand


undergone angioplasty?
a. Very high added risk grade 2 HPN

75. Metabolic syndrome (+) BP 160/100


High added risk grade 2 HPN

76. Smoker, call center agent (+) BP 150/90


Low added risk grade 1 HPN

77. Hx of CV dse. BP 170/100


Very high added risk grade 2 HPN

78. BP of 140/100 and HR of 110/min (+) asthmatic and


businessman. What will you prescibe to the patient?
CCB
COMMUNICABLE DISEASES SAMPLEX
November 2020

1. A 30 year old street vendor has acute onset of fever, c) S.enteritidis


malaise, and generalized body pain which started 5 d) S.choleraesuis
days prior to consult. He has difficulty on ambulation
because of severe pain on both legs. On PE he is 9. Which of the following infection causes pre- and
jaundice, with conjunctival congestion and muscle post-herpetic neuralgia?
tenderness. The standard serologic test to be a) HSV1
requested to rule in weils disease is b) Varicella
a) ELISA c) EBV
b) PCR d) HSV2
c) LAAT
d) MAT 10. Communicability of SARS2 peaks on?
a) Late in the incubation period or pre symptomatic
2. Optimal antibiotic for multi-drug resistant S.typhi period
includes? b) Second week of illness
a) Chloramphenicol c) After infection
b) Azithromycin d) First few days of illness
c) Amoxicillin
d) Trimethoprim-sulfamethoxazole 11. Which of the following is the most common cause of
sporadic viral encephalitis?
3. Which of the following is correct in low risk exposure a) EBV
to leptospirosis during flooding? b) Varicella
a) Wading with open skin lesion c) HSV1
b) Advice hospitalization d) HSV2
c) Doxycycline 100mg 2 cap single dose
d) Continuous exposure to contaminated water 12. Base on WHO HIV clinical staging HIV infected patient
presenting with Chronic thrombocytopenia (<50,000)
4. Leading causes of mortality with advanced HIV disease belong to:
globally is? a) Stage 1
a) Tuberculosis b) Stage 2
b) Sever bacterial infection c) Stage 3
c) Toxoplasmosis d) Stage 4
d) Pneumocystis pneumonia
13. Which the following vaccine can be safely given among
5. Base on WHO HIV clinical staging HIV infected patient HIV patients
presenting with Acute necrotizing ulcerative stomatitis, a) Varicella
gingivitis, or periodontitis is already in: b) Pneumococcal
a) Stage 1 c) MMR
b) Stage 2 d) Oral typhoid vaccine
c) Stage 3
d) Stage 4 14. Nasopharyngeal carcinoma risk is increase with
infection cause by which of the following?
6. Community acquired MERS are link to exposure to a) HSV2
a) Camel b) HSV1
b) Palm civet c) EBV
c) Horseshoe bat d) Varicella
d) Chicken
15. A 30 year old street vendor has acute onset of fever,
7. According to WHO HIV clinical staging HIV infected malaise, and generalized body pain which started 5
individual with manifestation of HIV encephalopathy is days prior to consult. He has difficulty on ambulation
classified to be in: because of severe pain on both legs. On PE he is
a) Stage 1 jaundice, with conjunctival congestion and muscle
b) Stage 2 tenderness. This case is unlikely to be typhoid fever
c) Stage 3 because of the presence of
d) Stage 4 a) Conjunctival congestion
b) Malaise
8. Which of the Non-typhoidal salmonella can also cause c) Body pain
similar illness with S.typhi d) Jaundice
a) S.paratyphi B
b) S.paratyphi A
COMMUNICABLE DISEASES SAMPLEX
November 2020

16. Which of the following is correct in acute retroviral 24. This technique that is done when examining rabid
syndrome? patients can detect the virus in fresh samples of saliva,
a) Not communicable skin, CSF and brain tissues. At the same time, it can
b) Manifest in all HIV patients distinguish the varied rabies virus variants:
c) Influenza or infectious mononucleosis like illness a) Direct fluorescent antibody testing
d) Usually 2 to 4 months after infection b) Rabies virus specific antibodies
c) PCR
17. Which of the following statements is correct about d) RT PCR amplification
rose spot?
a) Salmon colored 25. A 32 year old male presents with chills, fever, and
b) Exclusively in S.typhi anorexia. Examination shows an enlarged spleen. For a
c) Often in the extremities definitive diagnosis of this condition, what must be
d) Appear on the 3rd week of illness seen in the stained peripheral smear?
a) Antibodies to the parasite
18. Standard antiretroviral drug for treatment of naive b) Gametes
patients consist of? c) Asexual forms of the parasite
a) Single INSTI d) Sexual forms of the parasite
b) Dual PI
c) Triple ARV, 1 from any of the ARV group 26. A 54 y/o street sweeper is bitten by a stray dog in the
d) Dual NRTI plus 1 from other group of ARV thighs and abdomen. She presents to the ER with fever,
malaise, anorexia and vomiting. She also feels
19. Confirmatory test for COVID19 paresthesia at the wound site. What phase of the
a) Isolation test clinical stage of rabies is this?
b) Antigen test a) Prodrome
c) Serologic test b) Incubation period
d) NAAT c) Encephalitis
d) Paralytic
20. Person with close contact with a patient suffering from
covid19 should be quarantine within the duration of 27. In cases of malaria acquired through blood transfusion,
the incubation period which is up to? this is absent thus explaining the short incubation
a) 28 days period:
b) 7 days a) Pre-erythrocytic phase
c) 14 days b) Asexual stage
d) 21 days c) Erythrocytic phase
d) Sexual stage
21. What is the usual CSF examination result of a patient
who was bitten at the facial area by a rabid dog? 28. This eye finding is pathognomonic of military
a) Mild mononuclear cell pleocytosis tuberculosis in 30% of cases
b) Bloody non traumatic aspirate a) Choroidal tubercles
c) Normal CSF b) Corneal caseation
d) Normal protein level c) Lenticular ulcerations
d) Vitreal tubercles
22. Malaria during pregnancy caries a significant risk of
morbidy and mortality. Levels of parasitemia are 29. A 38 yo female patient residing near the Sierra Madre
highest and failure to clear the parasites after mountain range was often brought to the hospital as a
treatment is most frequent among primigravids. What child because of bouts of fever and chills. Her fever
medicines are safe to give to pregnant patients with pattern was described as quartan. She is now
malaria for all trimesters? diagnosed with nephrotic sydnrome which is this
a) Mefloquine and artemether specie of Plasmodium. What is the histopathologic
b) Mefloquine and quine appearance of this renal condition?
c) Quinine a) Dense deposits in the endothelium
d) Chloroquine and mefloquine b) Glomerulosclerosis of the subendothelial layer
c) Segmental glomerulonephritis with the splitting
23. What factors are associated with a poor prognosis in of the capillary basement membrane
tetanus? d) Diffuse glomerulonephritis with splitting of the
a) Diastolic blood pressure more than 90mmHg capillary basement membrane
b) Systolic blood pressure less than 140mmHg 30. Hypoglycemia is an important and common
c) Incubation period more than 7 days complication of malaria. This is due to several
d) Incubation period less than 7 days mechanisms cause by the parasite. However,
COMMUNICABLE DISEASES SAMPLEX
November 2020

hypoglycemia can be an adverse effect of this drug d) Non-specific immune response and vascular
which is used widely for uncomplicated and severe endothelium response
falciparum malaria: 37. A 35 y/o male teacher is assigned to train his fellow
a) Lumefantrine teachers living in Turkey. You must prescribe him with
b) Quinine a prophylactic medicine to protect him against malaria
c) Mefloquine while he is in that country for one month. He informs
d) Chloroquine you that he is allergic to doxycycline. What medicine
can you prescribe aside from doxycycline?
31. A 65 year old male matient, exposed to his wife who a) Chloroquine
was diagnosed to have PTB comes to your clinic b) Fansidar
because of low grade fever and cough. DSSM shows c) Lumefantrine
M.tuberculosis bacteria. He reports that he has not d) Primaquine
taken any anti TB drugs. What drug regimen should 38. A 56 yo male farmer develops tetanic spasms and is
you give to this patient? brought to the ER. What is the best drug to control
a) 2 months HRZE/4 months HR these spasms?
b) 56 days streptomycin and ethambutol a) Cetirizine
c) 2 months HRZE/10 months HR b) Diphenhydramine
d) 3 months HRZE/ 4 months HR c) Chlorpromazine
d) Potassium infusion
32. The two forms of rabies that eventually may 39. The two genera in the Rhabdoviridae family contain
deteriorate to moratlity are: species that cause disease to humans. This is the virus
a) Paralytic and comatose that affects a broad range of animals and can cause
b) Spinal and paralytic serious neurologic sequelae when transmitted to
c) Encephalitic and spinal humans:
d) Encephalitic and paralytic a) Vesiculovirus
b) Lapsavirus
33. Aside from a DSSM, this examination may also be c) Vermicolor virus
requested by a physician to definitely diagnose a d) Lyssavirus
patient with tuberculosis:
a) CT scan with contrast 40. True statement regarding Mild Leptospirosis:
b) Nucleic acid amplification test Conjunctival suffusion is a pathognomonic feature
c) Chest xray
d) CT scan with contast 41. Which sexually transmitted agent causes tropical
spastic paresis and T-cell Leukemia?
34. Liver and spleen enlargement, ascites, and wasting are a) HTLV-1
characteristically seen in this type of schistosoma?
a) S. Mekongi 42. A 22 y/o male college student sought consult at the ER
b) S. Hematobium due to a low grade fever for 5 days accompanied by
c) S. Mansoni body malaise, headache and joint pains. He admitted
d) S. Intercalatum to have had unprotected sex witha commercial sex
worker 1 week prior this consultation. Upon
35. Cardiovascular instability in severe tetanus is difficult examination, the medical intern noted blisters which
to manage. Rapid fluctuations in BP and HR may were itchy and painful on the lateral aspect of his penis
happen. This drug has been helpful in improving with clear urethral discharge and tender inguinal
cardiovascular stability by increasing sedation: lymphadenopathy
a) Magnesium sulfate IV a) We should increase the efficiency of transmission
b) Calcium IV infusion by promoting use of contraceptives
c) Captopril
d) Metoprolol 43. Laboratory finding on patient with typhoid fever
except:
36. What are the two host responses that develop in a) Elevated liver enzymes
patients with tuberculosis about 2-4 weeks after b) Leukopenia and neutropenia
infection? c) Positive stool cultures on the 1st week of illness
a) B cell mediated response and tissue damaging d) Bon marrow culture with >80% sensitivity
response
b) Lymphocyte activating response and tissue
damaging response
c) Macrophage activating response and tissue
damaging response
COMMUNICABLE DISEASES SAMPLEX
November 2020

44. Which is not true regarding Post Diarrhea 1. Kissing disease is best described by which of the
Complications? following?
a) Campylobacter infection = Guillain - Barre a. Fever, pharyngitis, chylosis for 1-4 weeks
Syndrome b. Fever, pharyngitis, lymphadenopathy for
b) Giardia = Hemolytic Uremic Syndrome 1-4 weeks
c) Chronic diarrhea = lactase deficiency c. Fever, pharyngitis, hepatomegaly lasting for
d) Salmonella = reactive arthritis 1 to 4 weeks
d. Fever, pharyngitis, splenomegaly lasting for
45. True statement regarding the prognosis of a patient 1 to 4 weeks
who had leptospirosis?
a) Majority of patients recover 2. Which of the following is correct in the
management of infectious diarrhea?
46. True statement regarding Schistosomiasis in the a. Mainstay treatment is adequate antibiotic
Philippines b. Empirical antibiotic for dysentery
a) Transverse myelitis is the second most c. Empirical antibiotic for patients with fever
problematic CNS problem d. Antimicrobial is a must if
enterohemorrhagic E.Coli is suspected
47. Which histopathologic finding is correctly matched
with a patient’s sign/symptom? 3. Regarding HIV infection the population that is least
a) Swelling with vacuolation= calf pain likely affected is:
a. Male having sex w/ male (MSM)
48. A 36 y/o MSM presented at the ER with fever, b. Commercial sex workers & client
generalized rashes not sparing the soles and palms c. Healthcare workers (HCW)
a) Aciclovir d. People who inject drugs (PWID)

49. True statement regarding praziquantel: 4. Infectious diarrhea associated with contaminated
a) The only available antiparasitic drug for egg is due to:
schistosomiasis a. E.coli
b. Shigella
50. Which is not true regarding the virulence factors c. Salmonella
utilized by microbes causing diarrhea? d. Vibrio cholerae
a) Neurotoxins may cause vomiting in patients after
ingestion of contaminated food 5. You have an emergency travel to a known P.
Falciparum malaria endemic area within 2 days.
51. Which of the following antibiotic has the correct dose Which of the following is your best choice of
for treating Salmonella prophylactic antimalarial drug?
a) Chloramphenicol 25mg/kg TID a. Primaquine - prevents drug resistant
falciparum and vivax malaria in adults
52. Which is not true in Proctitis b. Doxycycline
a) Presents as chronic diarrhea c. Mefloquine - for pregnant women WHO
53. Laboratory findings compatible with Leprospirosis (2nd and 3rd tri) for CDC (1st tri)
except: d. Choloroquine -can be used for prophylaxis
a) Leukocytosis with right shift and treatment

54. True statement regarding Traveler’s diarrhea 6. Which of the following is correct in transfusion
a) Enterotoxigenic E.coli is the single most common malaria?
important cause a. Primaquine is highly recommended
b. Clinical features is different from mosquito
55. A 22 y/o male college student sought consult at the ER borne
due to a low grade fever for 5 days accompanied by c. Longer incubation period
body malaise, headache and joint pains. He admitted d. None
to have had unprotected sex witha commercial sex
worker 1 week prior this consultation. Upon 7. A 22 y/o female came in due to dysuria and
examination, the medical intern noted blisters which frequency for 2 days. Urinalysis showed pyuria with
were itchy and painful on the lateral aspect of his penis bacteria of less than 100 cfu/ml. What is your initial
with clear urethral discharge and tender inguinal impression?
lymphadenopathy a. Acute urethral syndrome
a) Chancroid caused by H.ducreyi would cause a
painful genital lesion.
COMMUNICABLE DISEASES SAMPLEX
November 2020

b. Bacterial cystitis - w/ single patho (e.coli 14. This is the reason for the high concentration of the
and staph sapro) at counts of >1000 / ml of rabies virus in the saliva of an infected animal.
urine a. Viral reproduction happens in the salivary
glands
8. Diarrhea after eating raw sea foods is caused by b. Saliva constrains a lot of co-inhabiting
which of the ff EXCEPT? bacteria and viruses.
a. Vibrio specie c. Saliva is alkaline making the milieu more
b. Salmonella conducive to viral reproduction
c. Giardia lamblia d. Viral shedding from the sensory nerve
d. Hepatitis A endings in the oral mucosa

9. Which is not part of the management of typhoid 15. A 40 y/o patient from Kenya has a hx dysuria and
fever? hematuria. Circulating cathodic antigen is positive
a. Steroid therapy for complicated cases. in the urine. Lately he was complaining of nocturia,
b. Admit px with suspected salmonellosis dribbling and incontinence. The above case is
c. Surgery in chronic carries with anatomical caused by.
abnormalities (biliary stones) a. S. Mansoni
d. Chronic carrier may be managed with an b. S. Mekongi
appropriate antibiotic c. S. Japonicum
d. S. Haematobium

10. Infectious diarrhea associated with contaminated 16. A 40 y/o male farmer has bouts of melena and
egg is due to hematemesis. Ultrasound reveal periportal or
a. E. coli symmers clay pipestem fibrosis. Which of the
b. Shigella following is correct?
c. Salmonella a. His condition is more common in
d. Vibrio cholera adolescents
b. Due to egg-induced granulomatous
11. A 21 y/o male from Manila came to the ER due to responses -s.mansoni and japonicum
frequency loose bowel movements (5 episodes few c. Liver function is usually severely affected
hours prior to consult-mucoid, often blood d. Due to early inflammatory hepatosplenic
streaked). When asked by a medical clerk regarding schistosomiasis
his travel history, he said he attended a family
reunion in Cavite 2 days ago. Where they had some 17. A 22 y/o female patient came at the ER due to
crustaceans, fresh vegetable salad and fried rice. severe right upper quadrant abdominal pain.
His stool exam showed polymorphonuclear History revealed she had been febrile for 3 days
leukocytes. Which of the following could be most and had noted a moderate amount of vaginal
likely etiologic agent discharge for a week now. An impression of pelvic
a. Vibrio parahaemolyticus inflammatory disease was made by the IM resident.
b. Bacillus cereus - fried rice What would be your expected findings that would
c. Clostridium perfringens contribute to the diagnosis?
d. Staphylococcus aureus a. Whitish curd-like cervical discharge
b. Elevated SGPT, SGOT, Elevated WBC & ESR
12. Organism responsible for diarrhea associated with c. Negative pregnancy test, (+) fluid filled
abdominal cramps without fever caused by heat tubes seen via a transviganal utz
resistant spores in inadequately cooked gravy. d. Presence of “external dysuria” -acute
a. Enterotoxigenic E.coli urethral cys
b. Clostridium difficile
c. Bacillus cereus 18. A 25 y/o seafarer consulted the OPD with his
d. Campylobacter jejuni complaint of Urethral discharge which has been
going on for 8 days. He has no fever but complains
13. Hemolytic uremic syndrome in patient with of nontender lymphadenopathy in his bilateral
diarrhea is associated with inguinal areas and 1 cm ulcer with clean base at the
a. Hepatitis a inferior aspect of his penile shaft. What will you
b. EPEC E. coli request to confirm the diagnosis?
c. STEC E.coli a. Gram stain -LGV
d. Low mortality b. COPT-circumoval prescription test (schisto)
c. Tzanck smear herpes
d. Dark field microscopy syphilis
COMMUNICABLE DISEASES SAMPLEX
November 2020

25. Which statement is a feature of Inflammatory type


19. Patients with tuberculosis and HIV infections would of Diarrhea:
less likely have caviations in their chest X-ray. This a. Fecal mononuclear leukocytes are a feature
statement is: of Vibrio cholera - polymorphonuclear
a. False b. Shigella producing cytotoxin that allows the
b. True bacteria to invade the large intestine - small
c. It depends on the strain of tuberculosis intestine
bacilli c. Entamoeba histolytica secretes enterotoxin
d. It depends on the severity of HIV which produces characteristic stools -
Shigella
20. Your nephew who is studying in Manila, after d. Primarly affects the proximal small intestine
wading in the floodwaters for the first time came
home and asked you regarding post exposure
prophylaxis for leptospirosis which is true?
a. Doxycycline 200mg once a day for 3 days. 26. TRUE statement on Diagnostic methods for
-1-2days only Leptospirosis:
b. No need for prophylaxis since he has no a. Blood culture has high specificity and
cuts nor abrasions - still needed for low risk sensitivity
exposure b. Specimen for MAT is best collected
c. Doxycycline may be given within 72 hours - during >1 week of illness
LOW exposure 1 time flood (w/o c. A 2 fold rise in the MAT titer is confirmatory
cuts/wounds) of the diagnosis
d. He is classified as having moderate risk d. MAT is the gold standard of diagnosis
exposure MODERATE: one time flood; (+)
cuts/ wounds; (+) ingestion of water 27. TRUE statement regarding Praziquantel:
HIGH RISK: (+)continuous exposure; WITH a. Results in parasitology cure of <80% hence
OR WITHOUT cuts or wound the need for other treatment modalities
21. HP is positive for HIV and also present with chronic b. It is poorly absorbed and absorption is
thrombocytopenia; base on these the patient further decreased by food intake
belong to WHO HIV clinical stage: c. Shown to resolve hepatomegaly after
a. III therapy but not the fibrosis on liver
b. II parenchyma
c. I d. Has direct killing effect on Schistosoma
d. IV
28. What specimen is best collected on the 2nd and 4th
22. Diagnostic exam in Schistosomiasis which uses week of illness to aid diagnosis of leptospirosis?
schistosome eggs to detect antibodies in the a. CSF - 1st wk of illness
patient’s serum: b. Urine - 2nd to 4th wk
a. Kato katz - stool (Ascaris, Trichuris, c. Blood - > 1 wk
Hookworm, Schistosoma) d. Stool
b. ELISA
c. FECT 29. Patient is positive for HIV infection and blood
d. COPT chemistry revealed increase creatinine, proteinuria,
hypoalbuminemia, and hyperlipidemia; base on the
23. The leading cause of mortality in advance HIV information patient is in WHO HIV clinical stage?
disease is a. IV
a. Crytococcal meningitis b. III
b. Tuberculosis c. I
c. Pneumocystis pneumonia d. II
d. Severe bacterial infection

24. Which of the following is most likely an acute 30. True statement regarding Swimmer’s itch:
retroviral syndrome after two weeks of high risk a. Serum sickness-like illness brought about
exposure by commencement of oviposition by
a. Acute viral hepatitis like illness sexually maturing schistosomes
b. Acute meningococcemia like illness b. Rash may disappear quickly in patients
c. Acute influenza like illness exposed for the first time
d. None c. Major pathologic lesions are large
granulomas and fibrosis
COMMUNICABLE DISEASES SAMPLEX
November 2020

d. Commonly acquired 2-12 weeks after b. Ultrasound may show distended bladder
exposure with S.hematobium hydroureter or hydronephrosis
c. Urinary egg is abundant
31. True complication of Salmonellosis except: d. Cytoscopy may reveal sandy patches
a. Reversible psychosis
b. Neuropsychiatric symptoms 38. TRUE statement on management of Infectious
c. Hepatitis diarrhea:
d. Relapse with a different strain type a. ORS formulations containing rice or cereal
may not be as effective as glucose-based
32. The leading cause of mortality in advance HIV solutions
disease is b. IV fluid infusion is usually needed
a. Tuberculosis c. The current WHO recommendation states
b. Cryptococcal meningitis that 2.6gm NaCl is better than 3.5gm NaCl
c. Pneumocystis pneumonia as component of Oral rehydration
d. Severe bacterial infection solutions - dec. osmolarity; low stool
output, vomiting
33. TRUE statement on diagnostic methods for d. IV antibiotics is preferred over oral
Leptospirosis:
a. Blood culture has high specificity and 39. A 40 year old patient from Kenya has a history of
sensitivity dysuria and hematuria. Circulating cathodic antigen
b. Specimen for MAT is best collected is positive in the urine. Lately he was complaining
during >1week of illness of nocturia, dribbling and incontinence. Drug of
c. A 2 fold rise in the MAT titer is confirmatory choice for this patient is
of the diagnosis - 4 fold a. Praziquantel
d. MAT is the gold standard of diagnosis b. Diethylcarbamazine
c. Albendazole

34. This is included in the Mycobacterium complex and


is characteristically resistant to pyrazinamide: 40. A person positive for HIV presenting with
a. M.bovis generalized lymphadenopathy but denies of any
b. M.lepra other symptoms; what is his WHO HIV clinical
c. M.pyrabacterium stage?
d. M.cannonium a. IV
b. I
35. True regarding the use of Oral rehydration solution c. III
a. Reduced osmolarity/increased salt is better d. II
tolerated than classic ORS
b. Must be withheld in cases of Dysentery
c. Glucose-based ORS is more effective than 41. Which statement is a feature of Inflammatory type
formulations with rice or cereal of Diarrhea
d. May be given with breastmilk a. Fecal mononuclear leukocytes are a feature
of Vibrio cholera
36. A 32 year old male diabetic consulted because of b. Shigella producing cytotoxin that allows the
hemoptysis, low grade fever and chronic cough. bacteria to invade the large intestine
DSSM results were positive for Koch’s infection. c. Entamoeba histolytica secretes enterotoxin
What are the most important effector cells for which produces characteristic stools.
containment of this disorder? d. Primarily affects the proximal small
a. Phagocytes and lymphocytes intestine
b. Stabs and TNF
c. Lymphocytes and eosinophils 42. Main adaptive mechanism of shistosomes that
d. Phagocytes and eosinophils allow them to survive in humans for a long period
of time:
37. A 40 year old patient from Kenya has a history of a. Heptalaminar membrane
dysuria and hematuria. Circulating cathodic antigen b. Enzymatic secretions coming thru
is positive in the urine. Lately he was complaining minipores on their egg shells
of nocturia, dribbling and incontinence. Which of c. Ability to resist antibiotics
the following is correct? d. Their unique regulatory mechanism for
a. Cystoscopy reveal hypertrophic bladder pairing
mucosa
COMMUNICABLE DISEASES SAMPLEX
November 2020

43. The risk of acquiring M. tuberculosis infection is


determined mainly because of:
a. Endogenous factors
b. Innate resistance
c. Exogenous factors
d. Both exogenous and endogenous

44. A 25 y/o seafarer consulted the OPD with his


complaint of Urethral discharge which has been
going on for 8 days. He has no fever but complains
of nontender lymphadenopathies in his bilateral
inguinal areas and a 1 cm ulcer with clean base at
the inferior aspect of his penile shaft/ What is your
primary etiologic agent?
a. Herpes
b. Chlamydia
c. Treponema
d. Klebsiella

45. Best biochemical prognosticator in severe malaria


is
a. Serum potassium
b. Blood lactate
c. Blood glucose
d. Serum creatinine

46. Main adaptive mechanism of schistosomes that


allow them to survive in humans for a long period
of time:
a. Heptalaminar membrane
b. Enzymatic secretions coming thru
minipores on their egg shells
c. Ability to resist antibiotics
d. Their unique regulatory mechanism for
pairing

47. True statement regarding Praziquantel:


a. Serum levels are increased by
glucocorticoids and carbamezipine
b. Increases lactate release - calcium
c. Increases ATP level of Schistosoma
d. Major mechanism of action of the drug on
Schistosoma is disruption of the parasite
tegument
DERMATOLOGY SAMPLEX
November 2020

1. What is the etiologic agent of Ecthyma d. Tumor


gangrenosum?
a. Proteus mirabilis 9. What type of cells can be found in the stratum
b. Staphylococcus epidermidis germinativum?
c. Staphylococcus aureus a. Cuboidal cells
d. Pseudomonas aeroginosa b. Dendritic cells
c. Squamous cells
2. Which of the following is responsible for the blister d. Columnar cells
formation in Bullous impetigo?
a. Fillagrin deficiency 10. Which of the following viral exanthems presents as
b. Keratolytic agent maculopapular lesions that coalesce on the face and
c. Epidermolytic toxin neck whereas lesions on the extremities remain
d. Ceramide deficiency discrete in which of the following disease?
a. Rubeola
3. What form of tinea pedis is due to the presence the b. Rubella
substances produced by tricophyton rubrum that c. Exanthem subitum
diminishes immune response and inhibits stratum d. Roseola infuntum
corneum turnover?
a. Intertiginous type 11. Which of the following diseases presents with a
b. Mocassin-type macule?
c. Vesico-bullous type a. Wart
d. Ringworm pattern b. Melasma
c. Molluscum contagiosum
4. Separation of the nail plat is known as: d. Pityriasis rosea
a. Onychorrhexis
b. Onycholysis 12. Crops of painful and tender bright-pink, dermal and
c. Onychomycosis subcutaneous nodules arising in clinically normal skin
d. Onychomadesis on the upper and lower extremities associated with
arthralgias, arthritis, fever, anorexia, and malaise noted
5. Arthritis and lymphadenopathy is seen associated after months of treatment with anti-leprosy drugs:
with the maculopapular rash in which of the following a. Dapsone syndrome
conditions? b. Erythema nodosum leprosum
a. Roseola infantum c. Delayed type hypersensitivity reaction
b. Rubella d. Lucio phenomenon
c. Rubeola
d. Exanthem subitum 13. Vesicles are defined as circumscribed epidermal
lesion containing fluid. Which of the following present
6. Which of the following organisms cause with vesicles?
dermatophytosis? a. Varicella
a. Pityrosporum ovale b. Melasma
b. Trichophyton rubrum c. Xanthelasma
c. Candida albicans d. Erythasma
d. Malassezia furfur
14. Ulcerative impetigo is also known as:
7. Honey-colored granular crusts are seen in which of a. Carbuncle
the following? b. Ecthyma gangrenosum
a. Non-bullous impetigo c. Ecthyma
b. Bullous pemphigoid d. Furuncle
c. Bullous impetigo
d. Ulcerative impetigo 15. Herpes zoster is due to the reactivation of which of
the following organisms?
8. A solid round palpable ellipsoidal lesion located in a. Varicella zoster virus
the dermis is known as a: b. Human herpes virus 6
a. Papule c. Herpes simplex 2
b. Cyst d. Herpes simplex 1
c. Nodule
DERMATOLOGY SAMPLEX
November 2020

16. Erythrasma, which presents as erythematous to


light brown, half moon shaped plague in the groin is 23. What is the most common site of focal vitiligo?
caused by which of the following organisms? - tinea a. Back
cruris b. Feet
a. Trichophyton rubrum c. Hands
b. Corynebacterium minutissimum d. Face
c. Epidermophyton flucossum
d. Candida albicans 24. Which statement is TRUE of Paget’s disease of the
breast?
17. What is the significance of a positive hair pull test? a. It responds well to corticosteroids
a. Telogen effluvium b. Both breasts are usually affected - unilateral
b. Androgenetic alopecia c. A palpable breast mass is always present
c. Trichotillomania d. The nipple may be retracted
d. Alopecia areata
25. What is the most common symptom of Pyogenic
18. What is the nail lesion equivalent to Auspitz sign of granuloma?
psoriatic skin lesions? a. Swelling
a. Nail pits b. Pain
b. Leukonykia c. Bleeding
c. Splinter hemorrhages d. Pruritus
d. Salmon spots
26. Which statement is TRUE regarding metastasis of
19. What is an indication for removal and biopsy of Basal Cell Carcinoma
nevus? a. The most common cause of death is CNS
a. A new pigmented nevus in a 10 y/o male metastasis.
b. A Spitz nevus in a 7 year old female b. The initial site of metastasis is the lung
c. Pigmented nevus with a surrounding c. Metastasis is usually due to neglect -
depigmented zone on a 17 y/o male incomplete excision
d. Nevi on the hairy scalp on a 50 y/o female d. 5-year survival rate after metastasis is 30%
with history of malignant melanoma
27. What is the nail lesion in childhood Atopic
20. What is the most important risk factor for skin Dermatitis?
cancer? a. Tachyonychia - rough nails
a. Skin color b. White to yellow discoloration of the nail
b. Genetic predisposition c. Nail pits
c. UV exposure d. Splinter hemorrhages
d. Chemical exposure
28. A 7 year old boy presented with a 7mm. Round
21. What is a differential diagnosis for Erythroplasia of reddish brown nodule on the right thigh which
Queyrat? appeared 1 month prior to consult. When the lesion is
a. Secondary syphilis pinched gently, a central deression is seen. What is your
b. Zoon’s balanitis diagnosis?
c. Basal cell cancer a. Acrochordon
d. Oral hairy leukoplakia b. Solitary keratoacanthoma
c. Intradermal nevus
22. Which of the following skin cancers has the poorest d. Dermatofibroma
prognosis?
a. Squamous cell cancer on the leg of a 52 y/o 29. A 30 year old female presented with a 1.5x2cm
male erythematous scaly plaque on the scalp for 3 month
b. Bowen’s disease on the cheek of a 80 y/o duration. She noticed that there is a scar and hair loss
male on the center of the lesion. There are no other
c. Malignant melanoma on a 25 y/o female associated signs and symptoms. What is your
arising from a Giant Congenital Nevus on the diagnosis?
back a. Alopecia areata
d. Nodulo-ulcerative BCC on the nose of a 70 y/o b. Psoriasis
female c. Discoid lupus erythematosus
DERMATOLOGY SAMPLEX
November 2020

d. Tinea capitis a. An ulcer within the leukoplakia indicates


malignancy
30. A 7 year old boy presented with a 7mm. Round b. Constant lip biting may cause cancer
reddish brown nodule on the right thigh which c. Atypical cells are found in 50% of leukoplakia
appeared 1 month prior to consult. When the lesion is - 10-20%
pinched gently, a central deression is seen. What is the d. Leukoplakia with red lesions have lower risk of
best management at this time? malignant transformation - highest risk
a. Excision
b. Laser ablasion 36. What condition may be associated with sudden
c. Topical corticosteroid eruption of multiple seborrheic keratosis? - SCC, BCC,
d. Observe and wait for spontaneous regression Increased weight (obesity)
a. Pustular Psoriasis
31. A 65 y/o female consulted because of black nodule b. Hyperthyroidism
with central ulcer on the nose of 1 year duration. She c. Exfoliative dermatitis
noted that the lesion started as a cluster of d. Diabetes mellitus
hyperpigmented papules which increased in size and
developed painless bleeding which eventually ulcerated. 37. A 52 y/o security guard consulted because of
What is your diagnosis? pruritic erythematous papules with areas of
a. Fibroepithelioma of Pinkus hyperpigmentation on both lower legs. On examination
b. BCC, nodular on both lower legs. On examination, you also noted
c. BCC, cicatricial - cheeks prominent varicose veins. Which of the following
d. BCC, pigmented - brown or black procedures will be help alleviate the patient’s
pigmentation in trunk dermatitis?
a. Injection of steroids
32. A 7 year old boy presented with a 7mm. Round b. Phototherapy
reddish brown nodule on the right thigh which c. Sclerotherapy
appeared 1 month prior to consult. When the lesion is d. Chemical peeling
pinched gently, a central depression is seen. What
diagnostic test should be done to confirm your 38. Which of the following papulosquamous skin
diagnosis disorders presents with a herald patch and Christmas
a. CBC tree pattern?
b. Anti-ds DNA a. Lichen planus
c. Skin biopsy b. Seborrheic dermatitis
d. ANA c. Psoriasis
d. Pityriasis rosea
33. A 55 y/o male with hepatobillary cancer consulted
because of appearance of erythematous patches on the 39. In a 7 month old infant with atopic dermatitis, the
trunk composed of multiple blood vessels with a central lesions are most likely to be seen in what areas?
arteriole with fine radiating blood vessels. What are a. Trunk
these lesions? b. Hands and feet
a. Hemangioma c. Cheeks Popliteal areas
b. Venous araneus
c. Telangiectasia 40. Which of the following statements about acne is
d. Cherry angioma TRUE?
a. Anti-androgen is the most effective treatment
34. Which of the following is a dermatologic b. Comedonal acne is best treated with
manifestation of Lupus Erythematosus, Scleroderma, antibiotics
and Dermatomyositis? - telangiectasia c. It tend to be more severe among females
a. Alopecia d. It may be aggravated by high glycemic diet
b. Reynaud’s phenomenon
c. Urticaria 41. Test for hepatitis is recommended for patients who
d. Oral ulcers are diagnosed with:
a. Pityriasis rosea
35. Which statement is TRUE regarding leukoplakia and b. Lichen planus
cancer? c. Seborrheic dermatitis
d. Psoriasis
DERMATOLOGY SAMPLEX
November 2020

c. Lesions tend to be generalized among infants


42. Which of the following are expected lesions in d. Pruritus may be absent
perioral dermatitis? - pustules and papules
a. Comedones 50. Vitamin D analogues such as calcipotriol are
b. Telangiectasia recommended for patients with:
c. Vesicles a. Psoriasis
d. Pustules b. Pityriasis rosea
c. Lichen planus
43. Which of the following medications is likely to d. Seborrheic dermatitis
cause photodermatitis? - sulfa drugs, thiazide,
tetracycline 51. What is a common cause of flaring of lesions in
a. Erythromycin guttate psoriasis?
b. Allopurinol a. Diabetes
c. Simvastatin b. High humidity
d. Tetracycline c. Infection
d. Exposure to dust or allergens
44. Which of the following is common among
highly-tensed and anxious individuals? 52. A 44 y/o female presented with erythematous
a. Lichen simplex chronicus papules and pustules with telangiectasia on the cheeks,
b. Dyshidrotic eczema nose, and around the eyes. She reported having a “red
c. Nummular eczema face” for almost 2 years which becomes worse when
d. Atopic dermatitis she drinks coffee. Which of the following topical
products may worsen her condition?
45. Accelerated epidermal cell cycle is the cause of a. Betamethasone cream
scaly lesions in: b. Metronidazole gel
a. Seborrheic dermatitis c. Clindamycin solution
b. Psoriasis d. Sulfur lotion
c. Lichen planus
d. Pityriasis rosea 53. Which of the following is an associated feature of
the atopic dermatitis?
46. In severe cases of pityriasis rosea, which of the a. Itch when sweating
following oral medications may be recommended? b. Keratosis pilaris
a. Antibiotics c. Flushing
b. Steroids d. Personal history of asthma
c. Antivirals
d. Antifungals 54. A patient who is allergic to hair dye should be
advised to also avoid: - Allergic contact dermatitis
47. Which of the following is TRUE regarding irritant a. Rubber slippers
contact dermatitis? b. Metal jewelry
a. Diaper dermatitis is an example c. Henna tattoo
b. Nickel is the most common cause d. Perfume
c. It is related to personal or family history of
atopy 55. Which of the following statements regarding
d. Affected patients have a high load of pustular psoriasis is TRUE?
Pityrosporum yeast a. It is caused by Staphylococcal infection - No
organism isolated
48. Which of the following drugs may exacerbate acne: b. It is associated with malignancy
a. Pregabalin c. It is common in childhood psoriasis
b. Dapsone d. It may be caused by steroid withdrawal
c. Prednisone - lithium, OTC pills
d. Erythromycin 56. A 20 y/o female presented with a recurrent,
intensely pruritic, deep-seated vesicles on the sides of
49. Which of the following statements about scabies is her fingers. What is the MOST likely diagnosis?
true? a. Allergic contact dermatitis
a. Crotamiton is the treatment of choice b. Nummular eczema
b. It may be spread by an animal vector c. Dyshidrotic eczema
DERMATOLOGY SAMPLEX
November 2020

d. Stasis eczema c. Cholinergic urticaria - sweating/ inc. Body


tempt
57. What structure is involved in Syringoma? d. Urticarial vasculitis
a. Sebaceous gland
b. Hair follicle 65. Which of the following skin cancers has the poorest
c. Melanocyte prognosis?
d. Eccrine gland - Sweat glands a. Superficially spreading melanoma on the back
in a 22 y/o female
58. What is the nail lesion in childhood Atopic b. Nodular melanoma on the cheek with
Dermatitis? cervical lymphadenopathy in an 80 yo male
a. Splinter hemorrhages - Psoriasis c. Acral-lentiginous melanoma on the plantar
b. Nail pits - Psoriasis area in a 40 yo Japanese female
c. White to yellow discoloration of the nail - (+) d. Lentigo maligna on the arm with a duration of
Onycholysis; Proximal Subungual 6 mo
Onychomycosis
d. Tachyonychia 66. Which of the following has the highest risk of
malignant transformation?
59. What is the significance of a positive hair pull test? a. Small-sized congenital nevus
a. Trichotillomania b. Dysplastic nevus
b. Alopecia areata - Exclamation point hair c. Cutaneous horn
c. Androgenetic alopecia d. Solar keratosis
d. Telogen efflavium
67. Which of the following has the highest rate of
60. What type of alopecia is seen in Syphilis? metastasis?
a. Nonscarring a. Bowen’s disease
b. Cicatricial b. SCC of the lip with a thickness of 4mm - 8mm
c. Scarring c. SCC in an old burn scar
d. Diffuse d. Nodular Basal cell cancer on the nose

61. What type of keratoacanthoma is associated with 68. What is the most common type of Basal Cell
immunosuppresion? Cancer?
a. Ferguson-Smith type - family hx (+) crater form a. Pigmented BCC
scar b. Nodular BCC
b. KA dyskeratoticum et segregans c. Superficial BCC
c. Eruptive type d. Rodent ulcer
d. KA centrifugum marginatum- peripheral
expansion 69. Which of the following nevi presents as a pinkish
slightly scaly papule?
62. What lesion is seen in the sign of Leser Trelat? a. Spitz nevus
a. Seborrheic keratoses b. Balloon cell nevus
b. Actinic keratoses c. Intradermal nevus
c. Dermatofibroma d. Halo nevus
d. Solar lentigenes

63. What are the yellowish-brown spots on the nail


matrix and nail bed in patients with Psoriasis? 70. Which of the following is TRUE regarding Cellular
a. Splinter hemorrhages Nevi?
b. Endonyx a. Majority of nevi are present at birth - present
c. Salmon spots - oil spots 1st yrs; increase in numbers in 2-3 decades
d. Leukonykia b. Nevi are more numerous on the non-exposed
areas of the body - exposed to sun
64. What type of urticaria presents as appearance of c. Nevi are not stable and undergo stages of
wheals when the skin is stroked? maturation and senescence
a. Factitious urticaria - trauma d. The average number of nevi in a person is 30. -
b. Adrenergic urticaria - emotional upset 40
DERMATOLOGY SAMPLEX
November 2020

71. A 65 yo female consulted because of a nonhealing 78. Which of the following are expected findings in
eczematoid patch on the right areola accompanied by perioral dermatitis?
retraction of the nipple. What is your diagnosis? a. Comedones
a. Erythroplasia of Queyrat - penis b. Vesicles
b. Nipple eczema - (-) nipple retraction c. Pustules
c. Bowen’s disease - erythrophagia d. Telangiectasia
d. Paget’s disease

72. Which of the following is TRUE regarding Mycosis


Fungoides?
a. It is synonymous with Cutaneous T-cell
Lymphoma
b. It is a slowly progressive cancer
c. Lymph nodes are affected initially before 79. Which of the following is an associated feature of
cutaneous involvement - fatal atopic dermatitis?
d. Metastasis occurs early resulting to high a. Personal history of asthma
mortality b. Flushing
c. Hyperlinear palms
73. What is a sign of cancer arising in leukoplakia? d. Intolerance to spicy foods
a. Erythema
b. Increase in size
c. Pruritus
d. Induration - localized

74. What is the mainstay in the management of


Urticaria?
a. Allergen prevention
b. Systemic steroids
c. Antihistamines
d. Hypoallergenic diet

75. Which of the following statements about acne is


TRUE? 80. What oral medication may aggravate asteototic or
a. Anti-androgen is the most effective treatment xerotic eczema? - statin
- isotretinoin a. Metfromin
b. Comedonal acne is best treated with b. Prednisone
antibiotics - rosacea c. Rosuvastatin
c. It tend to be more severe among males d. Allopurinol
d. It may be aggravated by high fat diet - high
glycemic 81. Which of the following papulosquamous skin
disorders improves with topical steroids but may be
76. Which of the following anti-acne drugs regulates worsened by systemic steroids?
sebum production? a. Psoriasis
a. Doxycycline b. Pityriasis rosea
b. Benzoyl peroxide c. Lichen planus
c. Adapalene - retinoids; exfoliator d. Seborrheic dermatitis
d. Isotretinoin
82. Vitamin D analogue is a topical treatment for:
77. Aside from the convex area of the face, which of a. Pityriasis rosea
the following is a common site of rosacea? b. Lichen planus
a. Eyes c. Seborrheic dermatitis
b. Ears d. Psoriasis
c. Scalp
d. Neck 83. Which of the following papulosquamous skin
disorders has a possible viral etiology?
a. Lichen planus
DERMATOLOGY SAMPLEX
November 2020

b. Psoriasis c. Erythema at the periphery of the lesion may


c. Pityriasis rosea be due to hypersensitivity reaction
d. Seborrheic dermatitis d. Palms and soles are invloved - not involved

84. A 26 yo female presented with multiple 91. Which of the following is pathognomonic of
wrythematous oval papules with collarette scaling dermatomyositis?
oriented in a Christmas tree pattern at the back. Which a. Procimal muscle weakness
of the following statements is TRUE about this b. Gottron’s papules
condition? c. Shawl sign
a. It heals spontaneously in 2 wks - 6 to 12 weeks d. Aota
b. Pruritus may or may not be present
c. Oral steroids should be avoided 92. Fungal abscess is known as
d. Sun exposure will worsen the lesions a. Kerion
b. Furuncle
85. Aside from fluorinated toothpaste, what is a c. Carbuncle
common cause of perioral dermatitis? d. Maliariapustulosa
a. Topical steroids
b. Lipstick 93. Enanthem in rubella presenting as minute red
c. Mouthwash macules on the soft plate and uvula in some patients as
d. Cosmetics the prodrome resolves before the exanthema appears
is known as
86. What is the implication of an early onset of a. Scotoma
psoriasis? b. Forscheimer spots - rubella
a. It will resolve by the age of puberty c. Bitot’s spot
b. It tends to involve other organs d. Koplik’s spot - rubeola/measles
c. It may be associated with an autoimmune
disorder 94. Which of the following is the principal cell of
d. It will have a less stable course epidermis?
87. Heliotrope is characterized by which of the a. Mast cells
following? - telangiectasia b. Melanocytes
a. Dilated capillaries c. Langerhan’s cells
b. Violaceous discoloration in the periorbital d. Keratinocytes
areas
c. Thickening of the skin on the face 95. Which of the following organisms cause tinea
d. Erythematous to violaceous papules and a. Epidermophyton flucossum - tinea pedis
plaques found on the metacarpophalangeal b. Aspergillus fumigatus
joints c. Histoplasma capsulatum
d. Candida albicans
88. Non medical management of Tinea unguium:
a. Amorolfine 96. Because complications of varicella are more severe
b. Nail avulsion in adolescnets and adults, which of the followin drugs is
c. Itraconazole recommended?
d. Terbinafine a. Tenofovir
b. Retrovir
89. Which of the following is true of herpes zoster? c. Famvir
a. Arthritis may develop in adults - arthritis d. Acyclovir
b. Lesions are polymorphic - varicella
c. Vesicles are uniform in size - clusters 97. Which of the following drugs are included in the
d. Lesions are flesh colored umbilicated papules - WHO MB regimen for the management og
molluscum contagiosum multibacillary leprosy?
a. Pyrazinamide
90. Which of the following are true of molluscum b. Thalidomide - erythema nodosum leprosum
contagiosum? (Jopling type 2)
a. Umbilicated vesicles are frequently grouped c. Clarithromycin
b. Mosaic pattern is seen in clusters of several d. Clofazimine - rifampicin & dapsone
vesicles
DERMATOLOGY SAMPLEX
November 2020

98. Langerhan’s cells are found in which layer of the d. keratinocytes


epidermis?
a. Stratum spinosum
b. Stratum dysjunctum
c. Stratum germinativum
d. Stratum lucidum

99. Onycholysis, nail pitting, subungual thickening and


oil ataining are seen in which of the following
conditions:
a. Proximal subungual onychomycosis
b. Psoriasis
c. Lichen planus

100. Koplik’s spots is pathognomonic of which of the


following infections?
a. Varicella
b. Rubella
c. Verruca
d. Rubeola

101. Circumscribed area of change in skin color


measuring less than 5mm is known as:
a. Macule - < 1 cm
b. Plaque - < 1cm raised lesion
c. Patch - > 1cm without elevation
d. Scale

102. Which of the following is true of tuberculoid


leprosy?
a. Madarosis - lepromatous
b. “swiss cheese” or dimorphic lesions -
borderline lepromatous (BB)
c. Immunity is strong and sufficient to effect
cure
d. Leonine facies - lepromatous

103. Black dots can be seen in which pattern of


infection in Tinea capitis?
a. Pustular type
b. Seborrheic dermatitis type
c. Non-inflammatory
d. Inflammatory

104. Thrombosed capillaries can be present on the


surface of some warts as:
a. Verrucous surface
b. Minute erythematous papules
c. Friable cauliflower-like mass
d. Black dots - bleeding points

105. Collagen type VII is produced predominantly by


which of the following?
a. Melanocytes
b. Fibroblasts
c. Adipocytes
ONCOLOGY SAMPLEX
November 2020

b. Physiologic reserve
1. Tumor marker for testicular cancer c. Gender
a. PSA - prostate cancer d. Age
b. CA 15-3 - breast cancer
c. AFP 7. Vinorelbine targets what phase of the cell cycle
d. CEA - colorectal cancer a. S
b. M
2. A 65 yo female presents with odynophagia and c. G1
weight loss. Endoscopy revealed an esophageal mass d. G2
and biopsy of the mass revealed adenocarcinoma. On
complete work up, patient was found to be stage II 8. The histopath/cytology revealed: Squamous cell
esophageal cancer but because of his co-morbids; carcinoma of the lung. Base on the CT scan results, the
uncontrolled hypertension & diabetes mellitus type II patient is a stage IV squamous cell carcinoma of the
and post myocardial infarction was found to be lung. If you are planning to give chemotherapy, which is
medically inoperable but is still ECOG 1. What would be the best protocol to give?
the standard next line of treatment for this patient? a. Docetaxel + Cisplatin
a. ChemoRT - combination chemotherapy & b. Paclitaxel + Cisplatin
radiation therapy as the initial tx c. Paclitaxel + Carboplatin
b. Radiation therapy - spares perioperative d. Gemcitabine + Cisplatin
morbidity
c. Chemotherapy 9. Most common side effect of bevacizumab seen in
d. Best supportive care clinical practice today
a. Arterial thromboembolism
Stage I & II - Surgery and then ChemoRT b. Hypertension
Stage III & IV - ChemoRT only c. Bowel perforation
d. Hemorrhage

3. Cancers possibly cured with high dose chemotherapy 10. In high risk settings, can be used prophylactically to
with stem cell support prevent the development of leptomeningeal disease
a. Choriocarcinoma and brain metastasis in acute leukemia and lung cancer
b. Multiple Myeloma a. Radiation therapy
c. Colon Carcinoma b. Surgery
d. Renal cell Carcinoma c. Biologic therapy
d. Chemotherapy
Cancers possibly cured with high dose
chemotherapy with stem cell support: 11. A physically fit PS 0, 61 year old male is seen by a
Breast cancer thoracic surgeon for a newly diagnosed
Gestational Trophoblastic neoplasia adenocarcinoma of the lung. Patient was found to be
Testicular cancer Stage II Non small cell lung cancer upon completion of
work up. Patient underwent Left upper lobectomy.
Patient comes to see you post-operatively. The next
step should be:
4. Mechanism of action of Bevacizumab a. Give adjuvant chemotherapy alone for 6 cycles
a. Inhibition of VEGF b. Give oral TKI for 6 cycles to avoid recurrence
b. Inhibition of bFGF c. Give adjuvant chemo-rt for 3months
c. Inhibition of EGFR - Erlotinib d. Observe and monitor every 3 months for 3
d. Inhibition of PLGF years.
Stage 1 & 2 NSCLC - Surgery + Adjuvant
5. Dose limiting toxicity of Oxaliplatin
chemotherapy
a. Myelosuppression
Stage 3 NSCLC - Chemotherapy +
b. Sensory neuropathy
Radiotherapy
c. Hemorrhagic cystitis
Stage 4 NSCLC - Cornerstone of
d. Emesis
management: Standard medical
management, pain medicatipns &
6. Major determinant of treatment outcome
chemotherapy for palliation
a. Co morbidities
ONCOLOGY SAMPLEX
November 2020

3cm lung mass located centrally and upper abdomen CT


12. Unless tumors grow adequate blood vessel supply, scan shows multiple nodules in the liver metastatic in
they can not grow beyond this size nature. PAtient is ECOG 1. No weight loss. What is the
a. 1-2 mm3 best procedure to recommend to this patient to
b. 5 mm3 diagnose cancer?
c. <1 mm3 a. Percuatneous fine needle aspiration
d. 1 cm b. VATS wedge excision
c. Bronchoscopy
13. Antiangiogenic agent: d. Sputum cytology
a. Alentumumab
b. Cetuximab 19. Tumors poorly responsive in advanced stages to
c. Bevacizumab chemotherapy
d. Panitumumab a. Multiple Myeloma
b. Bladder cancer
14. A 65 yo woman presented with intermittent c. Seminoma
constipation and diarrhea associated with abdominal d. Salivary gland carcinoma
cramping. Colonoscopy revealed a near obstructing
mass and biopsy revealed moderately differentiated 20. Most common initial symptom of spinal cord
adenocarcinoma. Patient underwent resection of the 5 compression:
x 6 cm colonic mass. Pathology revealed a moderately a. Bladder distention
differentiated carcinoma invading in to the subserosa b. Localized back pain
with tumor involving 5 out of 22 lymph nodes. Surgical c. Lower extremity numbness
margins were negative. CT scan of chest and abdomen d. Lower extremity weakness
showed no evidence of metastatic disease. After
recovery from surgery, base on standard treatment 21. As tumor population increases, there is increase in
recommendation, you would do drug resistant cells due to somatic mutations.
a. Observation & monitoring a. Cancer cell kinetics
b. Bevacizumab + FOLFOX b. Fractional cell-kill hypothesis
c. Chemotherapy with FOLFIRI c. Goldie-Coldman hypothesis
d. RT to prevent recurrence d. Natural Resistance hypothesis

15. Complication of myelosuppression 22. Characteristic of cancer cells, EXCEPT:


a. Anemia a. Defective tumor suppressor gene
b. Febrile neutropenia b. Overexpressed oncogene
c. Thrombocytopenia c. Contact inhibition - Loss
d. Pancytopenia d. Increased growth factor secretion

23. Most common cause of mortality of tumor lysis


16. An increase in size of existing lesion/mass by 15% syndrome:
from baseline a. Arrhythmia
a. Complete response - total disappearance of b. Hyperkalemia
cancer c. Hypocalcemia
b. Stable disease -decrease of <50% size d. Acute renal failure
c. Partial response - decrease by atleast 50%
d. Progressive disease 24. Angiogenesis is an important event in:
a. Anaplasia
17. Cancers derived from connective and mesenchymal b. Metastasis
tissues c. Mutation
a. Sarcoma d. Autonomy
b. Carcinoma - epithelial
c. Lymphoma - lymph nodes, spleen, thymus 25. A 26 yo woman presents with large abdominal
gland and bone marrow. mass associated with abdominal pain. She underwent
d. Leukemia - WBC emergency explore laparotomy because of obstructive
symptoms and was found to have a large colonic mass
18. A 65 yo woman, never smokes has increasing cough that has already perforated. Pre operative CEA was
& pleuritic chest pain. CT scan of the chest reveals a
ONCOLOGY SAMPLEX
November 2020

elevated. Work up revealed patient in Stage II colon


cancer. Standard recommendation post op should be 31. Which of the following carries the best prognosis
a. Chemotherapy with FOLFOX a. ER +, PR +, HER 2+
b. Observation & monitoring b. ER +, PR +, HER 2 -
c. Cetuximab c. ER -, PR -, HER 2 -
d. Radiation therapy d. ER -, PR -, HER 2 +

26. A 59 yo male presents with severe back pain & 32. A 59 yo male presents with severe back pain &
bilateral leg weakness for 3 days. He claims to bilateral leg weakness for 3 days. He claims to
experience electrical tingling sensation down his spine experience electrical tingling sensation down his spine
prior to experiencing weakness of his lower extremities. prior to experiencing weakness of his lower extremities.
Recommended test during breast cancer What is the most probable diagnosis of this patient
surveillance/monitoring given the presentation?
a. Tumor marker CA 15-3 a. Hypokalemia
b. Bone scan b. Chronic fatigue syndrome
c. Mammogram c. Spinal cord compression
d. CT scan of chest & abdomen d. Cerebrovascular disease (stroke)

27. This the desired response to cancer treatment if 33. A 59 yo male presents with severe back pain &
the goal is TO CONTROL the disease: bilateral leg weakness for 3 days. He claims to
a. Stable response - to palliate experience electrical tingling sensation down his spine
b. Complete response - to cure prior to experiencing weakness of his lower extremities.
c. Partial response Best imaging modality to confirm the diagnosis?
d. Progressive disease a. Blood test including electrolytes
b. CT scan with contrast
28. Which is TRUE statement about tumor cell growth: c. Xray
a. The lag phase is the phase of cancer detection d. MRI
because of its explosive growth - Log phase
b. Tumor cell growth is linear - Gompertzian
c. Tumors in the plateau phase should undergo 34. Characteristic finding in SVC syndrome, EXCEPT:
tumor-debulking surgery before a. Dilated neck veins
chemotherapy can be effective b. Widened mediastinum
d. Very big necrotic tumors are mostly in the c. Pulsus paradoxus
logarithmic phase. - Plateau phase d. Facial edema

29. Which systemic chemotherapy is cytotoxic? 35. A 58 yo woman with stage III breast cancer was
a. Cisplatin given out patient systemic chemotherapy with
b. Interleukins docetaxel, doxorubicin and cyclophosphamide and was
c. Monoclonal Antibodies also given ondansetron. The next day she comes to see
d. Tamoxifen you because she had 2 episodes of blood in her urine.
You are suspecting hemorrhagic cystitis. Which among
30. A 58 yo man presented with anorexia, fatigue & the medication she received is the most likely culprit for
vague right sided abdominal discomfort for 2 months. the hemorrhagic cystitis
History revealed patient is a diabetic for 18 yrs & a. Doxorubicin
hypertensive for 10 yrs. His PE was unremarkable b. Ondansetron
except for mild pallor. Colonoscopy revealed a non c. Cyclophosphamide (Ifosfamide & acrolein)
obstructing transverse colon mass. Patient underwent d. Docetaxel
explore laparotomy & resection of circumferential
necrotic & fungating mass. Further work up revealed 36. A 59 yo male presents with severe back pain &
liver metastasis. Plan is to give systemic palliative bilateral leg weakness for 3 days. He claims to
chemotherapy. Which would be the recommended experience electrical tingling sensation down his spine
regimen base on this patient’s profile prior to experiencing weakness of his lower extremities.
a. FOLFIRI What is the most common malignancy associated with
b. FOLFOC this condition
c. Folinic acid + 5FU a. Lymphoma
d. Capecitabine b. Lung cancer
ONCOLOGY SAMPLEX
November 2020

c. Colon cancer diagnostic procedure to request to confirm your


d. Breast cancer diagnosis?
a. Upper GI series
37. The following are reasons for effectiveness of b. Endoscopic ultrasound
combinations of chemotherapy, EXCEPT: c. Triphasic CT scan
a. Cytotoxicity to both resting and dividing cells d. Upper GI endocscopy
b. Compatible side effects
c. Maximizing same mechanism of action 43. A patient diagnosed with gastric cancer comes to
d. Prevention of resistant clones you presenting as gastric outlet obstruction, What is
your initial treatment of choice?
38. A 46 yo female presents with right breast mass a. Chemotherapy
7x6cm in size with palpable axillary lymph nodes. b. Supportive care
Biopsy of the breast mass revealed invasive ductal c. RT
carcinoma. Further work up reveals patient has breast d. Surgery
cancer stage III. What is the standard treatment for this
stage 44. True statements regarding treatment of small cell
a. Surgery followed by surveillance lung Ca (SCLC) EXCEPT:
b. Neoadjuvant chemotherapy followed by a. Prophylactic cranial irradiation is
surgery followed by surveillance recommended for all patients with SCLC
c. Surgery followed by adjuvant chemotherapy b. Concurrent chemo-radiation is the treatment
d. Neoadjuvant chemotherapy followed by of choice for limited disease
surgery followed by adjuvant chemotherapy c. The cornerstone of therapy for extensive
disease is chemotherapy without thoracic
39. A 58 yo woman with stage III breast cancer was irradiation
given out patient systemic chemotherapy with d. Long time survivors of SCLC are at high risk for
docetaxel, doxorubicin and cyclophosphamide and was developing second primary malignancy
also given ondansetron. The next day she comes to see
you because she had 2 episodes of blood in her urine. 45. A 50 yo lady comes to you with a clinical stage 3B
You are suspecting hemorrhagic cystitis. What is the breast cancer ER and PR positive and HER2neu negative.
chemical irritant causing the bladder irritation and Workup revealed no distant metastases. What is the
hemorrhage? best treatment for her at this point?
a. Acrolein a. Neoadjuvant chemotherapy
b. Mesna b. Neodjuvant hormonal therapy
c. Formalin c. Outright MRM
d. Prostaglandin d. Neoadjuvant RT

40. Which is not a goal of treatment for 46. True of colorectal cancer, EXCEPT:
chemoradiation in esophageal cancer treatment? a. Left-sided tumors are annular in growth and
a. To eradicate occult micrometastatic disease therefore seldom cause obstruction
b. To increase the rate of complete resection b. Proximal colon cancers are more common
with negative circumferential margin c. Majority is adenocarcinoma
c. To downstage the disease d. Endoscopy with biopsy is the best method of
d. To lower the rate of distant metastases diagnosis

41. What is the best adjuvant treatment for a 47. What is the recommended adjuvant treatment for
premenopausal breast cancer patient who is Breast Cancer stage II, premenopausal, ER and PR
node-positive and ER-positive? negative HER2neu-negative?
a. Chemotherapy + hormonal therapy a. Chemotherapy with Herceptin
b. Radiation only b. Hormonal therapy only
c. Hormonal therapy only c. Chemotherapy and hormonal therapy
d. Chemotherapy only d. Chemotherapy only

42. A 60 yo man comes to your clinic with complaint of 48. Patient comes to you complaining of dysphagia
recurrent epigastric pain over a year this time with initially to solids then to liquids for 3 months now.
associated weight loss. What is the single best What will be the best method for diagnosis?
a. Upper GI endoscopy
ONCOLOGY SAMPLEX
November 2020

b. Chest xray macrovascular involvement and no extrahepatic spread


c. Abdominal CTscan of the tumor. What is the best option for this patient?
d. Abdomonal ultrasound a. Liver Transplantation
b. Laparoscopic surgical resection
49. A patient underwent abdominoperineal resection c. Radiofrequency ablation of the tumor
for rectal cancer stage II. What is your next treatment d. Open surgical resection
of choice?
a. Adjuvant chemotherapy 55. True about treatment of hepatocellular Ca (HCC)
b. No further treatment Except:
c. Adjuvant chemo-radiation a. Liver transplantation may be considered for
d. Adjuvant radiation locally advanced HCC with moderate cirrhosis
b. Patients who have ascites are poor surgical
50. Recommended treatment for early pancreatic candidates
cancer: c. Systemic chemotherapy has limited survival
a. Sugery + chemo benefit for the patients with advanced
b. Surgery + RT disease
c. Surgery only d. Surgery offers the only potential cure for early
d. Surgery + chemo-RT HCC

51. A 45 yo bank manager consults you for an 56. Which of the following has the highest risk for
incidental finding of a solitary liver nodule about 1.2cm Tumor Lysis Syndrome?
by ultrasound. He is anxious that it is malignant. He is a. Breast cancer with elevated creatinine and
asymptomatic and has unremarkable physical uric acid and normal LDH
examination findings. How will you best work-up this b. Acute leukemia with elevated LDH, normal
patient? creatinine, and uric acid
a. Request for HBsAg and Anti HCV, serum AFP c. Small cell lung cancer with elevated creatinine,
and core needle biopsy uric acid and LDH
b. Request for HBsAg and Anti-HCV, tri-phasic d. Lymphoma with normal creatinine, uric acid
CTscan, and core needle biopsy and LDH
c. Request for HBsAg and Anti-HCV and
multi-phasic CT scan then core-needle biopsy 57. G for cell cycle
only if necessary a. Gap
d. Request for HbsAg and anti-HCV and initial b. Genesis
serum AFP then repeat serum AFP and c. Gompertzian
ultrasound every 3 month d. Growth

52. The following indicate poor prognosis in lung Ca, 58. Level of clinical detection
a.
Except: 105
b.
a. Generalized weakness & fatigue 109
b. Large cell histology c. 1012
d.
c. Female sex 1014
d. Weight loss >10% in 6 months
53. True about the management of non-small cell Ca 59. Malignancy that is mesenchymal in origin
(NSCLC), EXCEPT: a. Carcinoma
a. Best supportive care is an option for b. Sarcoma
advanced & metastatic disease c. Leukemia
b. Radiation is an alternative for medically unfit d. Lymphoma
patients
c. Sequential chemo-radiation increases toxicity 60. Best diagnostic procedure for diagnosing breast
but has better local control compared to cancer:
concurrent chemo-radiation a. Core Needle Biopsy
d. Surgical resection remains the treatment of b. Excision Biopsy
choice if cure is the goal. c. Fine Needle Biopsy
d. Incisional biopsy
54. A 60 yo male with hepatitis C is diagnoses with HCC
stage 1 Child’s Pugh B. Work ups revealed no
ONCOLOGY SAMPLEX
November 2020

a. Irinotecan
61. The most common cancer that cause Death in b. Oxaliplatin
Females.
a. Breast 71. Side effect of oxaliplatin
b. Lungs a. -
c. Ovarian b. Hypertension
d. Liver c. Neuropathy
d. Neutropenia
62. Most common site of spinal cord compression:
a. Cervical 72. True of esophageal CA?
b. Thoracic a. Onset after 60 years old
c. Lumbar b. Common in High Socioeconomic Status
d. Sacral c. More common in Female
d. Uncommon but Lethal
63. Has propensity for bilaterality
a. Invasive Ductal CA 73. Most common site of esophageal cancer
b. Invasive Lobular Ca - Lower third
c. Ductal CA in situ
d. Lobular CA in situ 74. Most common hematologic spread of gastric cancer
a. Bones
64. Classification of doxorubicin b. Brain
- Topoisomerase Inhibitor c. Lungs
d. Liver
65. Breast cancer with best prognosis
a. ER/PR (+) 75. Standard Treatment for Stage 1 gastric cancer
b. ER/PR (-) a. Total gastrectomy with lymph node resection
c. HER2 (-) b. Subtotal gastrectomy with lymph node
d. Triple negative resection
c. Near total gastrectomy
66. Most common presentation in male breast cancer d. Subtotal gastrectomy only (**Ba’t kailangan
a. Nipple discharge ng Lymph Node Resection sa Stage 1?)
b. Peau d’orange
c. Unilateral gynecomastia 76. Most common presenting symptom of Pancreatic
d. Ulceration CA
- Abdominal Pain
67. Most common in lung cancer
a. Cough 77. Which of the following is a serum tumor marker for
b. Chest pain HCC?
c. Dyspnea a. AFP
d. Weight Loss b. CA 19-9
c. CEA
68. Treatment for stage 1 non small cell lung cancer d. 15-3
a. Surgery
b. Chemo RT 78. Treatment of choice stage IV HCC?
c. RT Sorafenib
d. TKI
79. High fat, low calcium and fiber. High risk for the ff
69. Premalignant polyp with high risk for colon ca cancer except
a. Pedunculated a. Pancreas
b. 2 cm polyp b. Colorectal
c. Tubulovillous c. Breast
d. Sessile d. Liver

70. Cornerstone Treatment for Colon CA: 80. Not a warning sign of malignancy
a. 5-FU a. Spotting in postmenopausal
b. Folinic Acid b. Nagging cough in smoker
ONCOLOGY SAMPLEX
November 2020

c. 1 cm mole with irregular border b. Chemoresistant but radiosensitive


d. Anemia due to bleeding c. Extrapulmonary (4%)
d. Peripheral in location
81. Prevention of Ca in person who does not have the e. All are true
disease.
a. Primary 90. A patient underwent abdominoperineal resection
b. Secondary for stage 2 rectal cancer. What is the treatment of
c. Tertiary choice?
d. Quarternay a. Adjuvant Chemotherapy
b. Adjuvant Chemotherapy-Radiation
82. Secondary prevention to Ca, except: c. Adjuvant-Radiation
a. Pap smear d. No further treatment
b. Mammography
c. Hep B vaccine 91. Recommended treatment for early pancreatic
d. DRE cancer
a. Surgery only
83. Response if the goal is to palliate b. Surgery + Chemo
- Stable c. Surgery + RT
d. Surgery +ChemoRT
84. Chemotherapy given after a locoregional
therapy…..? 92. Risk Factor for esophageal cancer
a. Primary therapy a. Menetrier's disease
b. Neoadjuvant therapy b. Caustic ingestion
c. Adjuvant therapy c. H. Pylori infection
d. Concurrent chemotherapy d. Previous PUD surgery

85. All are systemic cancer therapy, EXCEPT 93. Complaining initially dysphagia to solids then to
a. Cytotoxic liquids 3 months now. Method of diagnosis?
b. Hormonal a. Abdominal ultrasound
c. Immunologic b. Upper GI endoscopy
d. Brachytherapy c. Chest x-ray
d. Abdominal CT scan
86. Decrease in size of the tumor in two diameters by
25% 94. Poor prognosis in Non-Small Cell Lung CA
a. Partial (**Decrease in size po yun tanong) Answer: Large Cell Lung CA
b. Progressive
c. Stable 95. In what type of cancer would you recommend
d. Complete chemotherapy with or without RT if the patient has
good survival status?
87. The following indicate poor prognosis in Lung a. Stage 4 lung cancer
Cancer except: b. Stage 3 unresectable liver cancer
a. Weight Loss >10% in 6 mos c. Extensive SCLC
b. Generalized weakness and fatigue d. Locally advanced pancreatic cancer
c. Large cell history
d. Female sex 96. criteria for diagnosing colorectal cancer except
a. CRC involving 2 successive generations
88. Management of NSCLC except b. one first degree relative
a. RT is an alternative Tx for medically unfit c. earliest age to diagnose is 60
patients d. all are the criteria for crc
b. Surgery is the curative goal
c. Sequential chemo-RT is more toxic but gives 97. Risk factor for esophageal Ca, except
better control than concurrent chemo-RT a. Tylosis
d. io da! b. GERD
c. Strictures from caustic injury
89. True of Small cell lung CA d. Menetrier’s disease
a. Less aggressive than non small cell
ONCOLOGY SAMPLEX
November 2020

98. Gastric cancer risk factor, except:


a. Celiac Disease
b. H. Pylori
c. Previous gastric resection
d. Gastric atrophy

99. Characteristic findings of SVC Syndrome except:


a. Dilated Neck Veins
b. Widened Mediastinum
c. Pulsus Paradoxus
d. Facial Edema

100. Most common initial symptom of spinal cord


compression
a. Lower extremity weakness
b. Localized back pain
c. Lower extremity numbness
d. Bladder distention

101. Most common cause of mortality in tumor lysis


syndrome.
a. Hyperkalemia
b. Acute Renal Failure
c. Hypocalcemia
d. Arrhythmia

You might also like